Download Rh positive

Document related concepts

Maternal physiological changes in pregnancy wikipedia , lookup

Auditory system wikipedia , lookup

Organ-on-a-chip wikipedia , lookup

List of medical mnemonics wikipedia , lookup

Transcript
№ з/п
1
2
3
4
5
6
7
8
9
10
11
12
1.
Зміст розділу
К-ть питань в
контрольну
15
10
5
10
5
5
8
10
7
6
10
9
Excitable tissues
CNS
АNS
Endocrine Physiology
Sensory Physiology
High Nervous Activity
Blood physiology
Cardiovascular Physiology
Respiratory physiology
Physiology of energy balance and thermoregulation
Digestive physiology
Renal and Urinary Physiology
1
2.
1
3.
4.
1
1
5.
1
6.
1
7.
1
8.
1
9.
1
Which type of movement is the net diffusion of water
down its own concentration gradient?
Which process is used to recycle portions of the plasma
membrane?
Membrane potential is measured in these units
What is the ratio of sodium pumped out to potassium
pumped in by the active transport mechanism?
What is the function of carbohydrates in a plasma
membrane?
Osmosis
Carrier-mediated
Endocytosis
Exocytosis
Millivolts
3:2
Volts
9:1
Play an important Perform
role
in
"self" nonspecific
recognition
membrane
functions
The destruction of bacteria by a white blood cell is an Phagocytosis
Pinocytosis
example of
A group of cells was treated with a proteolytic (protein Diffusion of lipid Diffusion
of
digesting) enzyme Which of the following processes soluble molecules molecules through
would be least affected by the treatment?
through the cell membrane
membrane
channels
What of the following characterizes the internal Refers
to
the Is in a nonchanging
environment:
extravascular fluid state from moment
compartment
to moment
Exocytosis
Facilitated
diffusion
Dephosphorylation Facilitated
diffusion
Nanovolts
Kilovolts
3:1
5:2
Усього питань
153
113
47
103
56
26
52
101
35
21
88
75
Secondary
transport
Primary
transport
Decivolts
4:1
active
active
Form a barrier to Play role as a pump Play role as ion
diffusion
channels
Endocytosis
Egestion
Movement
of Attachment
of
molecules
by molecules
to
facilitated diffusion receptors in the cell
membrane
Is maintained by Refers
to
the
the interaction of intravascular fluid
special cells in the compartment
body
Which of the following statements concerns active Type of transport The greater the The
size
of The
rate
of
transport?
associated
with concentration
molecules
will diffusion can be
ATP support
gradient, the faster effect the way of altered by changing
the
rate
of transport
the temperature
diffusion
Active transport
Movement
of
molecules
by
active transport
Refers
to
the
intracellular fluid
compartment
Type of transport
non-associated
with ATP support
10.
1
11.
1
12.
1
13.
1
14.
1
15.
1
16.
1
17.
1
18.
1
19.
1
20.
1
Which one of the following is true of active transport?
Maintains
the Relies
upon
sodium-potassium facilitated diffusion
pump
Which of the following terms or phrases says that the Нypotonic solution Dynamic
amount of water making up a solution outside the cell is
equilibrium
greater than the amount of water making up a solution
inside of a cell?
What of the following is true of diffusion?
It is the tendency It is the movement
for
solute of substance by
molecules to move active transport
from an area of
high concentration
to an area of low
concentration
Movement through a membrane in response to pressure Filtration
Facilitated
difference is called?
transport
If cells are placed in a hypertonic solution containing a The cells will The cells will swell
solute to which the membrane is impermeable, what shrink
and
ultimately
could happen?
burst
Molecule "a" is transported from outside the cell
membrane where its concentration is 10% into the cell
where its concentration is 25% What has occurred?
Cells communicate with each other and with the outside
world through their:
What separates the intracellular fluid from the
extracellular fluid in humans?
The cellular membrane is made up of:
Active transport
Osmosis
Cell membrane
Pseudopodia
Relies on passive Donnan effect
diffusion
Osmosis
Osmotic pressure
Solute
Hypertonic
solution
It is the product of Maintains
the Movement
of
random movement sodium-potassium molecules
by
of
all
atoms, pump
active transport
molecules,
and
ions in a solution
Lysis
Active transport
The cells will
shrink at first but
will later reach
equilibrium with
the
surrounding
solution and return
to
original
condition
Facilitated
diffusion
The cells will show Osmotic hemolysis
no
appreciable
change due to
adaptation
and
diffusion of solute
and solvent
Diffusion
Intracellular matrix Extracurricular
space
Plasma membrane Nuclear membrane Cell wall
Endoplasmic
reticulum
Proteins
and Intracellular
All nucleic acid Chondroitin sulfate
phospholipids
ground substance
molecules
"Active transfer across cell membranes" means that:
Energy is used in Materials
are Cell
membranes Is the tendency for
the process
transported down a are nonselective in solute molecules to
concentration
choosing what goes move from an area
gradient
in and out
of
high
concentration to an
area
of
low
concentration
When a material moves from an area of higher Diffusion
Active transport
Phagocytises
Equilibrium
concentration to an area of lower concentration, this
Osmosis
Filtration
Cytoskeleton
Cillia
Sugar and spice
and everything nice
E Is the product of
random movement
of
all
atoms,
molecules, and ions
in a solution
Osmosis
process is generally called:
In active transport, phagocytosis, and pinocytosis one Energy
thing is always necessary What is it?
The process that refers specifically to the movement of Osmosis
water molecules through a semi permeable membrane
from a higher water concentration to a lower water
concentration is:
The cellular membrane is made up of:
Proteins
and
phospholipids
Which of the following represents the correct sequence Protein molecule,
of relative sizes (in descending order)?
amino
acid
molecule,
water
molecule, oxygen
molecule,
hydrogen ion
Which of the following terms or phrases says that the Hypotonic solution
amount of water making up a solution outside the cell is
greater than the amount of water making up a solution
inside of a cell?
Gibbs-Donnan effect leads to:
Osmotic gradient
Water
Diffusion
Facilitated
diffusion
Filtration
A
concentration Dynamic
gradient
equilibrium
Active transport
Phagocytosis
Intracellular
ground substance
Amino
acids
molecule, oxygen
atom,
protein
molecule,
hydrogen
ion,
water molecule
Dynamic
equilibrium
all nucleic acid
molecules
Amino
acid
molecule, oxygen
atom,
oxygen
atom, hydrogen ion
Chondroitin sulfate Sugar and spice
and everything nice
Amino
acid Oxygen
atom,
molecule, protein protein molecule,
molecule, oxygen hydrogen
ion,
atom
water molecule
Osmotic pressure
Hypertonic
solution
Diffusible
ions
between 2 sides
will be equal
Several types of
lipids
Equal
Equal
passive Non-diffusible ions
concentrations of diffusion
between 2 sides
ions on both sides
will be equal
Transport proteins Ionic pump
Pores of about 05
microns
in
diameter
Pinocytosis
Emeiocytosis
Diffusion
21.
1
22.
1
23.
1
24.
1
25.
1
26.
1
27.
1
What is unique property of the cell membrane in Antigenic proteins
intercellular communication?
28.
1
29.
1
Which of the following terms explains self-degradation Autolysis
Phagocytosis
of the cell:
The period of time during which a cell membrane is Absolute refractory After-potential
completely insensitive to another stimulus is the:
period
period
30.
1
Which of the following statements most accurately Enzymatic
The end plate is
describes features of neuromuscular transmission?
degradation of the highly enriched in
transmitter
can electrically
terminate
excitable gates
transmission
31.
1
"Active transfer across cell membranes" means that:
32.
1
Depolarization of the cell is produced by
Materials
are
transported against
a
concentration
gradient
Inward
active
transport of Na+
Energy is not used
in the process
Inward diffusion of
Na+
Solute
Threshold principle Relative refractory Nonperiod
accommodation
period
Acetylcholine
Each muscle fiber Acetylcholine
causes
calcium contains multiple causes
chloride
channels to open as axon terminals
channels to open as
a
result
of
a
result
of
membrane
membrane
depolarization
depolarization
Cell
membranes Materials
are Depends on the
are non-selective in transported down a size
of
choosing what goes concentration
hemidesmosomes
in and out
gradient
Active extrusion of Outward diffusion Inward diffusion of
K+
of K+
chlorides
The diffusible ion that is most important in the
establishment of the membrane potential is:
How will be changed the membrane potential in
hypokalemia?
An increase in intracellular sodium concentration is
explained by?
K+
Na+
Ca2+
C1-
Decreases
Increases
Is not changed
Is changed a little
Membrane
depolarization
Inositol
triphosphate (IP3)
Protein kinase A
Three-dimensional
shape
Hydrated diameter
33.
1
34.
1
35.
1
36.
1
37.
1
38.
1
39.
1
40.
1
41.
1
42.
1
43.
1
44.
1
Which of the following releases the neurotransmitters?
45.
1
46.
1
47.
1
Which part of the neuron has the lowest threshold Axon hillock
potential?
Which part of the neuron directs the signal toward the Dendrites
cell body?
Which of the following segment is most important in Trophic segment
neuron regeneration?
An increase in sodium conductance is associated with The
end-plate
which of the following:
potential of the
skeletal
muscle
fiber
Which of the following characteristics of an axon is The
conduction
most dependent on its diameter?
velocity of its
action potential
Membrane excitability will be increased by the greatest Decreasing
amount by which of the following:
extracellular Ca2+
The resting potential of a nerve membrane is primarily Potassium
dependent on the concentration gradient of which of the
following?
Which of the following characteristics of a water- Lipid solubility
insoluble substance is most important in governing its
diffusibility through a cell membrane?
Repolarization of the cell during an action potential is Inward diffusion of
produced by:
chlorides
If a poison such as cyanide stopped the production of The movement of
ATP, which of the following transport processes would Na+ out of a cell
cease?
Which of the following statements provides the best Continues until the
explanations of the absolute refractory period of the membrane
cell?
potential returns to
the resting level
Synaptic vesicles
K+, Na+
Cannot
determined
in Repolarization
be
An increase
intracellular
calcium
concentration
The downstroke of The upstroke of the The
refractory The plateau phase
the skeletal muscle smooth
muscle period of the nerve of the ventricular
action potential
action potential
cell action potential muscle
action
potential
The activity of its The duration of its The overshoot of The magnitude of
sodium-potassium refractory period
its action potential its resting potential
pump
Decreasing
Increasing
Decreasing
Increasing
extracellular H+
extracellular K+
extracellular Cl–
extracellular Na+
Bicarbonate
Sodium
Calcium
Chloride
Molecular weight
Electrical charge
Inward diffusion of Active extrusion of Outward diffusion Inward
active
Na+
K+
of K+
transport of Na+
Osmosis
The movement of The movement of Diffusion
K+ out of a cell
Na+ inside a cell
Is due to the high Occurs only during Occurs only during
negative polarity of the repolarization the depolarization
the inside of the phase
phase
cell
Axon hillock
Dendrite
Synaptic cleft
Axon terminal
Cell body
Collaterals
Occurs
during
depolarization and
the first part of the
repolarization
phase
Subsynaptic
membrane
Dendrites
Axon
Nodes of Ranvier
Axon hillock
Collaterals
Initial segment
Transmissive
segment
Conductive
segment
Receptive segment
48.
1
The depolarization of the neuronal membrane is Increases
the
produced by any factor that does one of the following:
membrane's
permeability
to
sodium ions
An acetylcholine blocking drug that acts on the Curare
postsynaptic membrane is
Which of the following is responsible for the falling The
sodiumphase of an action potential?
potassium atpase
pump restores ions
to their original
location
Which of the following statements concerning the Saltatory
propagation of an action potential characterizes the conduction occurs
myelinated nerve fibres?
in
myelinated
nerve fibers
49.
1
50.
1
51.
1
52.
1
Which of the following potentials relies to an “all-or- An action potential
none law”?
53.
1
54.
1
55.
1
Which of the following would occur in excitatory An action potential
synapse?
in the presynaptic
neuron
always
causes an action
potential in the
postsynaptic
neuron
The sodium-potassium pump is used to:
Get sodium ions
from inside of the
cell to outside
The period of time during which a cell membrane is Absolute refractory
completely insensitive to another stimulus is the:
period
56.
1
57.
1
58.
1
Inhibits
the
membrane's
permeability
to
sodium ions
Chlorpromazine
Stimulates
the Increases
the Decreases
sodium-potassium membrane's
membrane's
pump
permeability
to permeability
potassium ions
calcium ion
GABA
Homotropine
Propananol
the
Sodium gates open
The
increased The
increased The
increased
permeability
to permeability
to permeability
to
sodium
calcium
chlorides
Conduction
by
local current flow
is the method of
propagation
in
myelinated fibers
The
action
potential
jumps
from one Schwann
cell to an adjacent
Schwann cell in a
myelinated fiber
to
During conduction
by local current
flow, there is a
flow of current
between the active
and
adjacent
inactive area of the
cell membrane
An
excitatory An
inhibitory A
summation A
grand
postsynaptic
postsynaptic
potential
postsynaptic
potential
potential
potential
There is increased A
small Release in GABA Block of Ca2+ –
permeability of the hyperpolarization
channels
in
subsynaptic
occurs
presynaptic
membrane to both
membrane
sodium
and
potassium
Move
potassium Resting polarity
ions to the outside
of the cell
Threshold principle After-potential
period
Saltatory
conduction
is
slower
than
conduction by local
current flow
Move
chlorides
ions to the outside
of the cell
Relative refractory
period
Move calcium ions
inside of the cell
Nonaccommodation
period
During an action potential, repolarization begins as a Potassium
ions Potassium
ions Sodium
ions Sodium
ions Chlorides
ions
result of:
diffusing to the diffusing to the diffusing to the diffusing to the diffusing to the
outside of the cell inside of the cell outside of the cell inside of the cell inside of the cell
membrane
membrane
membrane
membrane
membrane
+
+
2+
+
+
+
2+
3+
When a neuron is at rest, the inside of the cell contains K ; Na
Ca ; Na
Na ; K
Fe ; Fe
Na+; Ca2+
more than
What force(s) best regulate the passage of sodium ions Sodium
and Electrical charge Sodium/potassium Osmosis
Filtration
across a nerve cell membrane?
59.
1
60.
1
61.
1
62.
1
63.
1
64.
1
65.
1
66.
1
67.
1
68.
1
69.
1
70.
1
potassium
ion
concentration
gradients
In order to have production of an action potential, the Decrease from -90
inside of the nerve cell must:
to -60 mV
How is the resting membrane potential of -70 MV The inside of the
(Millivolts) best interpreted?
fiber is 70 mV
more negative than
the outside
The primary site of protein synthesis in neurons is the
Cell body
on both sides of the pumps
cell membrane
Decrease from -70 Increase from -70 Decrease from -70
to -30 mV
to -90 mV
to +30 mV
The inside of the The outside of the The outside of the
fiber is -70 mV
fiber is -70 mV
fiber is 70 mV
more negative than
the inside
Axon
Dendrite
Terminal boutons
The entire period after an initial stimulus when a neuron Refractory period
Resting period
is non-sensitive to another stimulus is the:
In what way does the interior of a resting neuron differ Negatively charged Positively charged
from the external environment? It (the interior) is:
and contains less and contains less
sodium
sodium
The initial event that allows an action potential to occur Depolarization
The repolarization
is:
of the cell body
A period of time during which no stimulus will evoke a
response is called the:
The force of large skeletal muscle contraction depends
on the:
Absolute refractory Threshold phase
period
Amount of calcium Amount
of
ions
adenosine
triphosphate
present
Which of the following statements characterize the Is the period during Coincides with the
absolute refractory period?
which the other duration of the
stimulus cannot be action potential;
elicited;
A period of time during which no stimulus will evoke a Absolute refractory Threshold phase
response is called the:
period
The force of large skeletal muscle contraction depends Amount of calcium Amount
of
on the:
ions
adenosine
triphosphate
present
In muscle contraction, calcium apparently acts to:
Release
the Increase the action
inhibition on the potential
myosin filaments transmitted along
so that they can the sarco-lemma
interact with the
actin
Repolarization
Depolarization
Negatively charged
and contains more
sodium
The opening of the
pores due to a
stimulus
or
a
neurotransmitter
Summation time
Positively charged
and contains more
sodium
An influx of K+
Saltatory period
Increase from +30
to -70 mV
The inside of the
fiber is 70 mV
more positive than
the outside
Presynaptic
vesicles
Transient period
Negatively charged
and contains more
chlorides
An out pouring of
Na+
Relative refractory
period
Number of motor Rapidity of nerve Amount of sodiun
units within the impulses reaching ions
muscle
that the muscle
function
Continues until the Begins at the end Depends on the
membrane
of
the
single stimulus strength
potential returns to twitch;
the resting level;
Summation time
Saltatory period
Relative refractory
period
Number of motor Rapidity of nerve Amount of sodiun
units within the impulses reaching ions
muscle
that the muscle
function
Decrease the action Makes the bones Increase the action
potential
stronger so they potential
transmitted
can stand the pull transmitted
of the muscles
71.
1
72.
1
73.
1
74.
1
75.
1
76.
1
77.
1
78.
1
79.
1
80.
1
81.
1
82.
1
83.
1
84.
1
The strength or force of a muscle contraction in response The number of The intensity of
to different levels of stimulation is determined by:
motor
units stimulation
of
stimulated
individual muscle
fibers
The action of acetylcholine to increase the permeability Receptor sites on Synaptic vesicles
of the muscle fiber is stopped when:
the
fiber
are open
occupied by an
antibody
During contraction of a myofibril, which of the The length of the The length of Afollowing remain at their original length?
sarcomere
zone
Each of the following statements about skeletal muscle Its function is
is true except one Identify the exception
largely
under
involuntary control
During short periods of intense exercise, energy in Creatine phosphate
muscles is primarily derived from:
and
anaerobic
respiration
A condition in which stimuli occur so rapidly that there Complete tetanus
are no intervening relaxations is called:
Athletes usually experience muscular fatigue less Make
more
quickly than nonathletic because of a greater ability to
efficient use of
ATP
When a cell's membrane is said to be "less negative" it Relatively positive
means that it is ? Compared to the previous state.
The action potential arriving at the end of an axonal Precisely identical
membrane is:
to the initial action
potential
After firing an impulse, a neuron is incapable of Absolute refractory
responding to another stimulus for the duration of its:
period
Inactivation of the sodium-potassium pump will cause An increase in the
which of the following?
intracellular
volume
An increase in sodium conductance is associated with The
end-plate
which of the following:
potential of the
skeletal
muscle
fiber
Which of the following characteristics of an axon is The
conduction
most dependent on its diameter?
velocity of its
action potential
Membrane excitability will be increased by the greatest Decreasing
The number of
fibers
which
respond in a motor
unit
Cholinesterase is
inhibited
The characteristics The duration of
of each muscle stimulation
of
group
individual muscle
fibers
Acetylcholinesteras Phosphorylation by
e is bound at synapsine
receptor sites
The length of I- The
distance
zone
between M and Zline
It attaches to bones It is responsible for It produces more
of the skeleton
locomotion
and heat than any other
facial expression
tissue
Aerobic respiration Anaerobic
Creatine phosphate
respiration
Refractory period
Convert
glucose
into lactic acid
Relatively negative
Multiple
wave
summation
Tolerate
high
concentrations of
carbon dioxide
Absolutely
negative
Moderately weaker
than the initial
action potential
Threshold level
Greatly
weaker
than the initial
action potential
Membrane
potential
An increase in the Hyperpolarization
flow of sodium out of the membrane
of the cell
potential
Incomplete tetanus
The
distance
between
A-zone
and Z-line
It is composed of
fibers that appear
to be striated
Aerobic respiration
and
aerobic
respiration sources
Treppe
Carry on aerobic Make less efficient
respiration
use of ATP
Less positive
Absolutely positive
Moderately greater
than the initial
action potential
Action potential
Equal to the resting
membrane
potential
Resting period or
potential
An increase in the
intracellular
potassium
concentration
The plateau phase
of the ventricular
muscle
action
potential
The magnitude of
its resting potential
An increase in the
excitability
of
nerve cells
The downstroke of The upstroke of the The
refractory
the skeletal muscle smooth
muscle period of the nerve
action potential
action potential
cell action potential
The activity of its The duration of its The overshoot of
sodium-potassium refractory period
its action potential
pump
Decreasing
Increasing
Decreasing
Increasing
85.
1
86.
1
87.
1
88.
1
89.
1
90.
1
91.
1
92.
1
93.
1
94.
1
95.
1
96.
1
97.
1
98.
1
amount by which of the following:
The resting potential of a nerve membrane is primarily
dependent on the concentration gradient of which of the
following?
Which of the following characteristics of a waterinsoluble substance is most important in governing its
diffusibility through a cell membrane
Which one of the following muscle proteins plays an
important role in contraction of both smooth and striated
muscle
In a nerve, the magnitude of the action potential
overshoot is normally a function of which of the
following
The velocity of nerve conduction is increased with a
decrease in the which of the following
extracellular Ca2+
Potassium
extracellular H+
Bicarbonate
extracellular K+
Sodium
extracellular Cl–
Calcium
extracellular Na+
Chloride
Lipid solubility
Three-dimensional
shape
Hydrated diameter
Molecular weight
Electrical charge
Actin
Myosin
chains
Tropomyosin
Calmodulin
Extracellular
sodium
concentration
Capacitance of the
nerve
fiber
membrane
Which of the following would cause an immediate Increasing
reduction in the amount of potassium leaking out of a (hyperpolarizing)
cell
the
membrane
potential
Excitation-contraction coupling in smooth muscle is Calmodulin
initiated when calcium binds to which of the following
Snaptic transmission between pain fibers from the skin Substance P
and spinal cord neurons is mediated by which of the
following
When skeletal muscle is in its resting state, myosin Tropomyosin
cross-bridges are prevented from binding to actin
molecules by which of the following
Compartments within a muscle fiber separated by "Z" Sarcomeres
lines are called
Creatinephosphate functions in the muscle cell by:
Storing
energy
which
can
be
transferred to ADP
to
resynthesize
ATP as needed
Which one of the following factors does not influence Response to be
contraction?
produced
Which of the following factor that decreases muscle Increase in blood
fatigue?
flow to a muscle
The all-or-none principle, applied to muscle means:
Diameter
axon
light Troponin
of
the Intracellular
potassium
concentration
Resting membrane Degree
potential
myelinization
Decreasing
the
potassium
equilibrium
potential
Protein kinase A
Resting membrane Magnitude of the
potential
stimulus
of Space constant of Diameter of
the nerve fiber
nerve fiber
Increasing
intracellular
potassium
concentration
Troponin
Serotonin
Increasing
the
permeability of the
membrane
to
potassium
Myosin
light
chains
Acetylcholine
Endorphins
Somatostatin
Phospholamban
Calmodulin
Troponin
Titin
Fasciculi
Motorunits
Motor neurons
Forming
a
temporary
chemical
compound
with
myosin
Strength
of
stimulus
Reduction in blood
flow through a
muscle
Individual muscle Individual motor
Forming
chemical
compound
actin
a Inducing
conformation
with change
in
myofilaments
Length of muscle
fiber
Reduced muscular
energy reserves
Muscles
the Reducing
the
activity of the
sodium-potassium
pump
Tropomyosin
Multiunit smooth
muscles
a Forming
a
chemical
the compound
with
troponin
Weight or load to Duration
be moved
stimulus
Respiratory
Reduction
disorders
reserves
contract Muscles
the
of
of
ca
contract The whole muscle
fibers
contract units contract in a
completely or not graded
fashion,
at all
that
is
from
minimal
to
maximal
Immobilization of Muscle hypertonus
the muscle being
innervated
A motor neuron Connective tissue
and the muscle covering around a
fibers it stimulates muscle
with
stimuli
minimal by summation of contracts
rather
stimuli
than single motor
units
99.
1
Destruction of any part of the motor unit leads to:
100.
1
The term "motorunit" is applied to:
101.
1
Cholinesterase is found in association with:
102.
1
In response to depolarization,
reticulum of a muscle fiber:
103.
1
Each of the following statements about skeletal muscle
is true except one Identify the exception
104.
1
When an action potential reaches the terminal end bulb
of a motor neuron?
105.
1
Creatine phosphate serves to:
106.
1
107.
1
108.
1
109.
1
To accomplish a strong contraction, are stimulated at a
rapid rate?
Athletes usually experience muscular fatigue less Carry on aerobic Convert
glucose Tolerate
high
quickly than nonathletes because of a greater ability to
respiration
into lactic acid
concentrations of
carbon dioxide
A muscle that has been flaccid for a long time may Atrophy
Greater size
Greater tone
exhibit:
If nerve impulses cannot reach a skeletal muscle then:
Contraction of the The muscle dies The individual dies
muscle will not immediately
immediately
occur
110.
1
the
sarcoplasmic
Hyperkinesis
The union of a
muscle tendon with
the periosteum of
bone
The motor end The sarcoplasmic The skeletal system
plate
reticulum
Releases calcium Bonds to myosin
Bonds to actin
ions
into
the
sarcoplasm
Its function is It attaches to bones It is responsible for
largely
under of the skeleton
locomotion
and
involuntary control
facial expression
Acetylcholine
is Calcium
ion Activation
of
released
diffuses into the cholinestherase
muscle fiber
Supply energy for Cause
the Cause
the
the synthesis of decomposition of decomposition of
ATP
ATP
ADP
Many motor units Muscle cells
Whole muscles
The initiation of the mechanism that allows the actin Ca2+
filaments to slide across the myosin is done
"immediately" by which of these:
K+
Na+
Muscle spasm
Treppe
The triad in a The
myoneural
skeletal
muscle junction
fibers
The sarcolemma as The
endocrine
a whole
system
Releases potassium Depolarizes again
ions
It produces more
heat than any other
tissue
Inhibition
of
cholinestherase
It is composed of
fiber that appear to
be striated
Activation of Mcholinreceptors
Supply energy for
the change of ATP
to ADP
Muscle tones
Supply energy for
the change of ADP
to AMP
Myofibriles
Make
more Use
of
efficient use of atp creatinphosphate
Hypercalcemia
Hypocalcemia
The muscle will Epp will not be
contract as usual produced
since it has its own
innate
impulse
producer
Acetylcholine
Action potential
111.
1
112.
1
113.
1
114.
1
115.
116.
1
1
117.
1
118.
1
119.
1
120.
1
121.
1
The accumulation of which of these substances in the Lactic acid
muscle is considered a factor in producing muscle
fatigue?
Antagonistic action of muscles involves the:
Contraction of one
member of a pair
of
opposing
muscles and the
relaxation
and
stretching of the
other member
Myoglobin is a protein with a special function in muscle It holds a reserve
tissue because:
supply of oxygen
in the muscle
Oxygen
Glucose
Sodium
bicarbonate
Adenosine
triphosphate
ADP molecules on the surface of actin serve as active
sites for the formation of cross bridges with molecules
of:
ATPase activity is associated with:
The kind of contraction where tension rises to a
maximum but the length of the muscle remains constant
is called:
The type of contraction represented by a single
stimulus/contraction/relaxation sequence is called
Which of the following will contribute to muscle
fatigue?
Myosin
ATP
Activated myosin
Isometric
F-actin
Complete tetanus
G-actin
Treppe
I-aint-actin
Isotonic
Ca iones
Incomplete tetanus
Twitch
Complete tetanus
Incomplete tetanus
Isometric
Treppe
Fatigue of one Relaxation of both Contraction of both Action
of
the
member of a pair members of a pair members of a pair biceps brachii and
of muscles
of muscles
of muscles
the biceps femoris
It breaks
glycogen
down It
causes
the It produces the end It
provides
release of calcium plate potential
creatinphosphate
ions
from
the
sources
sarcoplasmic
reticulum for use in
releasing
the
myosin from actin
Troponin
Tropomyosin
None of the above
Decreased amounts Increased amounts Increase of ATP Ability of
the
of calcium ion in of calcium ion in reserves
motor neuron to
the sarcoplasm
the sarcoplasm
produce sufficient
quantities
of
acetylcholine
Which of the following will contribute to atrophy:
Results when a Involves
an Is characterized by Occurs
more
limb
is increase in muscle an increase in frequently
in
immobilized for a fibers
muscle size
younger
long period
individuals
who
exercise frequently
A contraction of a whole muscle in response to a single A
twitch Rigor mortis
Multiple motor unit Isometric
stimulus that causes an action potential in one or more contraction
summation
contraction
fibers, is called:
The stimulus for action potential generation in a skeletal A single motor Two
sensory A single sensory At least 2 separate
muscle is provided by:
neuron, or one of neurons
neuron
motor neurons
its branches
Increased ability of
the motor end plate
to
respond
to
stimulation
Causes an increase
in blood flow to the
affected muscle
Isotonic
contraction
The
autonomic
nervous system
122.
1
123.
1
124.
1
125.
1
126.
1
127.
1
128.
1
129.
1
130.
1
131.
1
132.
1
133.
1
134.
1
135.
1
Factors affecting the force of contraction include:
Degree of muscle Nature
of Cross-sectional
stretch prior to the neurotransmitter
area
of
the
contraction
muscle/muscle
fibers
Which of the following is a characteristic of skeletal Found in the calf Found in digestive Under control of
muscles:
and
hamstring tract
automatic nervous
muscles
system
Which of the following is a true statement?
The basic unit of Voluntary muscles Skeletal muscles
muscle contraction contract
more contract
more
is
called
a slowly
than slowly than cardiac
sarcomere
visceral muscles
muscles
The major purpose of the sarcoplasmic reticulum is to:
Store and then Store K+
Accumulate
2+
release Ca
calcium
The ability of muscle tissue to return to its original shape Elasticity
Excitability
Extension
after contraction or extension is called:
Which of the following factors will depend on the Rigidity
Contractility
Excitability
depletion of ATP-sources?
Process of anaerobic respiration is associated with:
Produces
lactic Is used during Produces the most
acid within the sustained periods atp
for
each
muscle
during of exercise
glucose molecule
fatigue
used
Which substance represents a back up storehouse of Creatine phosphate ATP
ADP
energy for muscle contraction?
Process of hypertrophy is associated with:
Is characterized by Involves
an Results when a
an increase in increase in the limb
is
muscle mass
number of muscle immobilized for a
fibers
long period of time
such as when it is
in a cast
"Oxygen debt" is defined as the:
Oxygen required to Amount of oxygen Amount of oxygen
remove lactic acid owed
to
the needed for quiet
which accumulates respiratory system respiration
after
strenuous
exercise
Without normal stimulation on a regular basis, muscles Atrophy
Hypertrophy
Hypertonus
will?
A continuous contraction that shows no evidence of Spasm
Fatigue
Isotonic
relaxation is called:
contraction
Identify the condition in which the muscle fiber only Incomplete tetanus Refractory period
Multiple motor unit
partially relaxes between contractions:
summation
An individual has just ingested a chemical that binds Sudden irreversible Irreversible
No contraction at
Emotional state of Length
the individual
muscle
of
the
can't be influenced Fiber has only one
at will
nucleus per cell
The origin of a Skeletal muscles
muscle is its distal contract
more
point of attachment slowly than smooth
muscles
Split ATP
Reabsorb
acetylcholine
Tetanus
Treppe
Elasticity
Spasm
Creatinphosphate
Oxygen
mechanism
Creatine
Lactic acid
debt
Occurs
less Causes an decrease
frequently
in in blood flow to the
younger
affected muscle
individuals
who
exercise frequently
Amount of oxygen Anaerobic
needed for the glycolysis
action
at
the
sarcomere
Summation
Treppe
Repolarization
Twitch
Complete tetanus
Treppe
Contraction
then Treppe
irreversibly to the acetylcholine receptors in the muscle contraction
sarcolemma The consequence to skeletal muscle would
be:
Immediately following the arrival of the stimulus at a Latent period
skeletal muscle cell there is a very short period called
the --?-- in which depolarization and repolarization of
the sarcolemma occurs
The function of the T-tubule is to
Carry the wave of
depolarization, or
charge, of the
sarcolemma
into
the fiber
Which of the following characterise the intensive muscle Tetanus
contraction?
The bacterium that causes tetanus produces a toxin that Competing
with
affects the central nervous system and skeletal muscles acetylcholine for
producing powerful tetanic contractions of the skeletal receptors at the
muscles The toxin probably acts by:
postsynaptic
membrane
Which of these answers characterise the propagation of Unmyelinated
nerve impulse by myelinated nerve fibres?
conduction
Nerve impulses "hop" from node to node in the process Saltatory
known as
conduction
ATP is important for a neuron so that,
The Na+/K+ pump
can operate
136.
1
137.
1
138.
1
139.
1
140.
1
141.
1
142.
1
143.
1
144.
1
145.
1
Unlike continuous conduction, saltatory conduction
146.
147.
1
1
A lack of adequate blood flow to active muscles
Neurotransmitters
Damage to a postsynaptic terminal would
Interfere with the
ability to respond
to neurotransmitter
Which of the following statements characterise the Provide a temporal
dendrites
and
spatial
summation
Requires
energy
contraction as soon all
relaxation
as the first nervous
stimulation reached
the lemma
Relaxation period
Absolute refractory Contraction period
period
Release
ions
Relative refractory
period
calcium Icrease
Hydrolyze (break) Break of amp
permeability of the atp
membrane
to
sodium
Twitch
Tonus
Treppe
Making
cell Competing
with Increasing
the
membrane
more acetylcholine for amount
of
permeable
to receptors at the acetylcholinesteras
calcium ions
presynaptic
e in the synapse
membrane
Nodes of Ranvier
Myelin sheath
Saltatory
conduction
Negative feedback The all or none Regeneration
principle
Outward diffusion Regeneration
of Inward diffusion of
of K+
CNS neurons can Cltake place
Increase the release Decrease
the Result in increased
of neurotransmitter release
of neurotransmitter
neurotransmitter
production
Arise from an area May be single or Vary in length
of a cell body branched
into from
a
few
called the hillock
collaterals
millimeters to over
1 meter
less Is a slower means
of
impulse
transmission
Ischemia
Hypoxia
Have
short-term Remain in the
effects
on the synaptic cleft for
Fatigue
Making the cell
less permeable to
sodium ions
Prolonged
conduction
Positive feedback
Inward diffusion of
Na+
Result in increased
rate of epp
Have
a
distal
portion
which
branches into an
arrangement called
telodendria
Requires
more Is characteristic of Does not have a
energy
unmyelinated
refractory period
neurons
Anoxia
Hypercapnia
Lactoacidosis
Blind irreversibly Are
always Diffuse
into
to the receptor catabolized
by presynaptic
postsynaptic
terminal
148.
1
Which of the following is true?
The nerve impulse
can travel only one
way
across
a
synapse
149.
1
Which of the following are connected with dendrites of Secrete
a neuron:
acetylcholine
and/or
norepinephrine
Why is it that the nerve impulse cannot be made to go The
dendrite
backward across the point of synapse, as from dendrites cannot produce a
to axons?
neurotransmitter
substance
150.
1
151.
1
Energy is conserved in saltatory conduction because:
152.
1
153.
1
If a neuron were experimentally placed in conditions The
intracellular
where intracellular potassium was depleted, which of the charge
would
following Is a logical consequence?
become
more
negative
Regarding the speed of impulse conduction, which of Myelinated fibers
the following is True?
conduct impulses
most rapidly
154.
2
Which of the following statements is/are correct
Effector organs are
non-neural
structures
155.
2
Which of the following is/are true
Action potentials
can
only
be
generated in the
axon hillock or
axon of neurons
Impulse
transmission does
not require energy
in
myelinated
neurons
long
time
periods
of molecules of the
postsynaptic
terminal
The
A synapse is the
neurotransmitter
junction between
secreted by all the afferent fibers
dendrites
is of two neurons
acetylcholine
Are not covered Conduct
graded
with a neurilemma action
potentials
(sheath
of toward the cell
Schwann)
body
The refractory state The
synapse
is too long
becomes fatigued
and
unable
to
transmit
the
impulse
acetylcholinesteras
e in the synaptic
cleft
The
electrical
activity
of
an
impulse raises the
threshold of the
synapse
Do not regenerate,
if cut
There is a physical
barrier
which
prevents
the
impulse
from
crossing at the end
of the dendrite
Depolarization
Depolarization
Depolarization
only has to occur in occurs only at the does not occur in
the axis cylinder nodes of Ranvier
cells with saltatory
down its entire
conduction
length rather than
in
the
myelin
sheathe
The neuron would The
intracellular The neuron would
hyper polarize
charge
would fire spontaneously
become
more
positive
Nerve
fibers Nonmyelinated
Motor impulses are
classed as pain fibers
conduct conducted
more
fibers conduct most impulses
most rapidly
than
rapidly
rapidly
sensory impulses
A reflex always The brain is the Afferent pathway
starts at a sensory functional unit of carries
impulses
neuron and ends at the nervous system from CNS
a motor neuron
Graded potentials Graded potentials Action
potential
are generated not are generated by can be generated in
only
at
the axons
any part of the
dendrites or cell
neuron
body of neurons
membrane
The terminations of
motor neurons on
muscle cells are
termed synapses
Two-way
conduction
nerve impulse
of
The CNS alone
produces
neurotransmitter
substances
Repolarization
does not occur in
cells with saltatory
conduction
Inward movement
of sodium
Nerve impulses all
travel at the same
rate
Efferent pathway
carries
impulses
toward CNS
Summation occurs
in axons
156.
2
157.
2
158.
2
159.
2
160.
2
161.
2
162.
2
163.
2
164.
2
Somatic reflex arcs:
Are the functional,
simplest units of
the central nervous
system capable of
receiving
a
stimulus
and
producing
a
response
Curarelike substances (dithylinum) make it impossible Neuromuscular
for skeletal muscles to contract because they block:
synapses
Are
the
basic Require conscious
structural units of thought for their
the central nervous initiation
system
Are
usually Effector
is
composed of one represented
by
or two, maybe neural structure
three neurons
Central synapses
It was established that the conduction velocity in the Motoneuron axons
nerve fibers was equal to 120 m/sec. Specify these
fibers:
Nerve impulses are termed:
Afferent if they
pass toward the
central
nervous
system
Preganglionic
sympathetic
Membrane
conduction
excitement
Postganglionic
sympathetic
The route or pathway of information to a reflex center, Afferent pathway
or the pathway from a receptor to an integrator,
describes an
Which of these statements is first out of order CNS sorts out or
concerning the basic functions of the nervous system?
integrates
incoming
nerve
impulses
Ganglionic
synapses
Preganglionic
parasympathetic
Efferent if they Reflexes when they
pass toward the pass back along the
cell body
same pathway they
follow in entering
the neuron cell
body
Motor nerve
Motor neuron
Proprioceptors
of
Postganglionic
parasympathetic
Afferent if they
pass both toward
and away from the
cell body
Afferent if they
pass along a neuron
away from the cell
body
Efferent pathway
Lowest
neuron
Conversion
of Conduction
of CNS
interprets
stimuli to nerve nerve impulses via nerve impulses
impulses in nerve sensory nerves to
centres
spinal cord and/or
brain
sensory
CNS routes nerve
impulses
to
effector organs, ie,
muscles,
and/or
glands, via motor
or efferent nerves
Which of the following would be correct to motor Efferent neurons Interneurons
Efferent neurons Efferent neurons Efferent neurons
neuron?
that
innervate (association
that
innervate that
innervate that
innervate
skeletal muscle
neurons) lie outside glands
blood vessels
internal organs
of
the
central
nervous system
Which class of neurons accounts for 95% of all nerve Association
Afferent neurons
Efferent neurons
Motor neurons
Ganglion network
cells?
neurons
A reflex arc:
Usually has
a Is the automatic Always includes a Always involves Always
involve
afferent, efferent, action
resulting receptor
and the spinal cord
cerebral cortex
and
association from
impulse effector
neurons
as conduction
to
components
muscle or gland
cells
Which of the following is not necessary for the
performance of the withdrawal reflex:
A neuron which has its primary function to connect
other neurons is called:
In a polysynaptic reflex, which neuron is located entirely
within the CNS?
A glial cell that is a major component of the blood-brain
barrier is the
Damage to a postsynaptic terminal would
165.
2
166.
2
167.
2
168.
2
169.
2
170.
2
171.
2
172.
2
173.
2
174.
2
175.
2
The summation of EPSP from numerous presynaptic
nerve fibers converging onto one postsynaptic neuron is
called:
Hyperpolarization of the postsynaptic membrane in
response to glycine or GABA is produced by the
opening of:
The excitation of the centre of one group of muscles (for
instance) which is accompanied with a simultaneous
inhibition of the antagonist group centres is related with:
Postsynaptic inhibition is produced by:
176.
2
The spread of excitation through the CNS is called:
177.
2
178.
2
179.
2
The inhibition perfoming by means of Renshaw cells is
called:
The circulation of nerve impulse through the closed
neurone chains is known as the:
The absent of spinal proprioreceptivereflexes are a sign
Cerebral cortex
Effectors (muscle Synapses
fibers)
association A motor neuron
A sensory neuron
Receptors
Peripheral nerves
An
neuron
Association
A glial cell
Schwann cells
Motor
Sensory
Ganglionic
Inhibitory
Oligodendrocyte
Astrocyte
Microglia cell
Ependymal cell
Schwann cell
Increase the release Decrease
the
of neurotransmitter release
of
neurotransmitter
The postsynaptic Excitatory
response can be postsynaptic
reduced
by potentials
are
presynaptic
generated
by
ingibition
opening channels
through which both
Na+ and K+ can
flow
Depolarization of Hyperpolarization
the
postsynaptic of the postsynaptic
membrane
membrane
Post-tetanic
Temporal
potentiation
summation
Result in increased
neurotransmitter
production
Summation
is
required to drive
the
postsynaptic
cell to threshold
Result in decreased
neurotransmitter
production
Inhibitory
postsynaptic
potentials
are
generated
by
opening channels
through which Clcan flow
Post-tetanic
potentiation
None of the above
Synaptic plasticity
Transformation of
the rate of impulses
Cl- gates
Na+ gates
Ca2+ gates
Mg2+ gates
Phenoumena
of
resiprocal
inhibition
Hyperpolarization
of the postsynaptic
membrane
Irradiation
Principle of final Principle
common path
feedback
Interfere with the
ability to respond
to neurotransmitter
The CNS and the neuro-muscular junction have in The
synaptic
common which of the following characteristics of transmitter agent is
synaptic transmission:
removed from the
synaptic cleft by
being
actively
transported into the
presynaptic
terminal
Presynaptic inhibition is produced by:
Axo-axonic
synapses
Spatial summation
Recurrent
inhibition
Reverberation
Depolarization of
the
postsynaptic
membrane
Principle
of
convergence
Resiprocal
inhibition
Convergence
The dorsal horn of The frontal lobe
K+ gates
of Plastisity of nerve Transformation of
centres
the rate of impulses
Axo-axonic
synapses
Post-tetanic
potentiation
Repolarization of
the
postsynaptic
membrane
of Reverberation
Occlusion
Presynaptic
inhibition
Summation
Principle
dominant
Pessimal inhibition Plastisity of nerve
centres
Divergence
Occlusion
The brainstem
The
cervical Any point that is
of a lesion of which portion of the nervous system?
180.
2
181.
182.
2
2
183.
2
184.
2
185.
2
186.
2
187.
2
188.
2
189.
2
the spinal cord or
corticospinal tract
any point distal to
this structure
In which part of spinal cord located centers for The cervical region The thoracis region The lumbal region The sacral part of
micturition and defecation?
of spinal cord
of spinal cord
of spinal cord
spinal cord
Which cell type is the predominant cause of the seizure Pyramidal cell
A burst of what type of potentials may initiate an Excitatory
epileptic seizure?
postsynaptic
potentials (EPSPS)
Which of the following chemical mechanisms could Γ-aminobutyric
underlie seizure generation
acid
(GABA)
inhibition
Which one of the following events accompanies the An increase in the
rapid voluntary flexion of the arm?
activity of the Ia
afferent fibers from
the triceps (the
antagonist)
A patient is diagnosed with a form of a peripheral A reduction in
neuropathy This individual will likely display which of conduction
the following
velocity of the
affected nerve
Basket cell
Inhibitory
postsynaptic
potentials (IPSPS)
Na+
channel
blockade
A decrease in the
activity of the Ib
afferent fibers from
the biceps (the
agonist)
Signs of an upper
motor
neuron
(UMN) paralysis
An increase in the
activity of the
alpha motoneurons
to the triceps (the
antagonist)
An increase in the
number
of
Ranvier's nodes
A decrease in the
activity of the Ib
afferent fibers from
the triceps (the
antagonist)
Degeneration
of
myelin but the
axon will typically
remain intact
During an in vitro experiment, the membrane potential
of a nerve cell is hyperpolarized to – 120 mV At that
time, a transmitter, known to be inhibitory in function, is
applied to the preparation and results in a depolarization
of the membrane Which of the following is the most
likely reason for this occurrence?
The passive spread of a presynaptic current acorss a gap
junction that is activated by changes in voltage, pH, or
calcium ion levels is most closely associated with which
of the following
Which of the following characterizes a principal feature
of axosomatic synapses
Inhibitory
transmitters
normally
depolarize
the
postsynaptic
membrane
Receptor potentials The
resting
potential
The
normal
response of the
postsynaptic
membrane to any
transmitter
is
depolarization
The
action
potential
Sodium channels
become inactivated
An increase in the
activity of the Ia
afferent fibers from
the biceps (the
agonist)
A loss in motor
function,
but
sensory functions
will remain largely
intact
The
inhibitory Calcium channels
transmitter
become activated
activates
ligandgated
potassium
channels
Electrical
postsynaptic
potentials
Purkinje cell
Membrane
potentials
Substance
inhibition
Stellate cell
Nernst potential
proximal to the
upper
cervical
spinal cord
Any point that is
proximal to the
upper
cervical
spinal cord
Schwann cell
Resting potentials
P Aspartate
inhibition
They are typically It is referred to as a They form the
inhibitory
type I synapse
predominant
synapse of cortical
projections to the
neostriatum
Which of the following is a second messenger system Adenosine
3',5'- Prostaglandins
Inositol
1,4,5directly activated by the binding of norepinephrine to a cyclic phosphate
triphosphate (IP3)
beta-adrenergic receptor
(camp)
Synaptic
transmission
mediated
glutamate
Glutamate
inhibition
Electrical
presynaptic
potentials
They
have
an
is electrical
by continuity linking
the
preand
postsynaptic cells
Diacylglycerol
Arachidonic acid
(DAG)
190.
2
191.
2
192.
2
193.
2
194.
2
195.
2
196.
2
197.
2
198.
2
199.
2
200.
2
201.
2
Hyperpolarization of the neuron is governed by which of
the following
The release of the transmitter is directly governed by
which of the following
N-methyl-D-aspartate (NMDA), kainate, and quisqualate
all act on which of the following receptors
In the biosynthesis of dopamine, which of the following
is the immediate precursor of dopamine?
Chloride
and Chloride
and Sodium only
potassium
sodium
Calcium influx
Sodium influx
Sodium efflux
Excitatory amino GABA receptors
acid receptors
LTyrosine
dihydroxyphenylal
anine (l-dopa)
Which of the following receptors requires the NMDA receptor
GABAA receptor
simultaneous binding of two different agonists for
activation?
Removal of norepinephrine from the region of the A combination of A combination of
synaptic cleft may be achieved by which of the enzymatic
enzymatic
following mechanisms
degradation,
degradation
and
diffusion,
and diffusion
reuptake
Which of the following types of inhibition have been Feed-forward,
Feed-forward and
identified within the dorsal column nuclei?
feedback,
and distal
inhibition
distal inhibition
only
Which one of the following reflexes is not likely to Vomiting reflex
disappear in a spinal animal:
The ascending tracts of the spinal cord include:
Corticospinal
Withdrawal reflex
Dopamine
receptors
Dopamine
hydroxylase
Potassium
and Sodium
and
sodium
calcium
Potassium efflux
Potassium influx
Adrenergic
receptors
- Tryptophan
Tyrosine
hydroxylase
L-AP4 receptor
Kainate receptor
AMPA receptor
Reuptake
Enzymatic
degradation
Diffusion
Feed-forward
inhibition utilizing
local interneurons
only
Stretch reflex
Feedback
inhibition utilizing
local interneurons
only
Flexor reflex
Distal
inhibition
from fibers arising
in the cerebral
cortex only
Urination reflex
Spinotectal
Spino-olivary
Spinoreticular
Anterior
spinothalamic
Vagal and sacral Vagal, sacral, and
efferent fibers only descending fibers
from the cerebral
cortex
Precentral gyrus
Basilar pons
Bladder functions are regulated by which of the Lumbar,
sacral,
following combinations of inputs:
and
descending
fibers from the
cerebral cortex
A college student received an injury as a result of being Nerve roots of C5tackled in a football game After the game, the student C6
was treated at a local hospital and was found to be
unable to abduct and rotate the left arm at the shoulder,
flex the elbow, and extend the wrist of the left side Upon
further examination, testing revealed depression of the
biceps reflex of this limb, but the reflex activity
involving the other limbs was normal Which of the
following is the most likely site of the injury?
A neurological examination of a 75-year-old male T8–T12
spinal C1–C5
reveals that when the abdominal wall is stroked, the segments
segments
muscles of the abdominal wall of the side of the body
stimulated failed to contract Other neurological tests
Opioid receptors
spinal C6–T1
segments
Lumbar and sacral Lumbar, thoracic,
efferent fibers only and cervical fibers
only
Ventral horn cells Triceps muscle
at C1
spinal T2–T7
segments
spinal L1–L5
segments
spinal
202.
2
203.
2
204.
2
205.
2
206.
2
207.
2
208.
2
209.
2
210.
2
211.
2
appeared normal Which of the following region is most
likely affected:
Which of the following pathways all cross in the spinal Anterior
cord:
spinothalamic tract,
lateral
spinothalamic tract,
anterior
corticospinal tract
Lateral
spinothalamic tract,
anterior
spinothalamic tract,
posterior
spinocerebellar
tract
The absent proprioceptive reflexes are a sign of a lesion The dorsal horn of The frontal lobe
of which portion of the nervous system?
the spinal cord or
any point distal to
this structure
Pathient's loss of left-sided pain and temperature Right
Right
fasciculus
sensation could be due to damage to which tract?
spinothalamic tract cuneatus
A pathient has difficulty in producing a smile from the Lower pons
Upper medulla
left side of his face, and he can't salivate or produce tears
from the left eye Further analysis showed some loss of
taste and that the affected muscles were flaccid and the
eyelids were open?
Immediately following transection of the spinal cord at Retention of urine A period of spinal
L2 in human beings there is:
and
faeces shock lasting for a
associated
with day
increased skeletal
muscles tone
Vital paralysis with hypotonia, bradycardia, dyspnoe, The
medulla The thalamus
dysphagia usually occurs with disorders in area, which is oblongata and pons
located in:
During a neuro-surgical operation the occipital areas of Visual
Tactile
cerebral cortex are stimulated. What sensations will the
patient have?
Systemic defence reflexes (coughing, vomiting, Medulla oblongata Midbrain
sneezing, and blinking) are controlled by areas in:
and pons
When a patient is asked to follow an object when it is Midbrain
Medulla
placed in the right side of his visual field, he is unable to
move his right eye either up or down The lesion is most
likely situated in which of the following:
A patient is capable of displaying pupillary constriction Pretectal area
Optic nerve
during an accommodation reaction but not in response to
a direct-light stimulus The lesion is most likely present
in which of the following:
Anterior
spinocerebellar
tract,
posterior
spinocerebellar
tract,
lateral
vestibulospinal
tract
The brainstem
Anterior
corticospinal tract,
lateral
spinothalamic tract,
dorsal columns
Medial
vestibulospinal
tract,
lateral
spinothalamic tract,
anterior
spinothalamic tract
The
cervical Any point that is
corticospinal tract proximal to the
upper
cervical
spinal cord
Right
fasciculus Left spinothalamic Left corticospinal
gracilis
tract
tract
Upper pons
Lower midbrain
Upper midbrain
A general increase Retention of urine Dyspnoe
in skeletal muscles and faeces
tone
The pineal gland
The pituitary gland The hypothalamus
Auditory
Olfactory
Gustatory
Cerebellum
Basal ganglia
Hypothalamus
Basilar aspect of Pontine tegmentum Cerebellum
the pons
Ventral
cell Visual cortex
column of cranial
nerve III
Optic chiasma
212.
2
213.
2
214.
2
215.
2
216.
2
217.
2
218.
2
219.
220.
2
2
221.
2
Damage to the subthalamic nucleus will result in which Hemiballism
of the following:
In Parkinson’s disease which out of the following Dopamine
transmitter is reduced in the basal gangia:
Torsion dystonia
Tremor at rest
Spastic paralysis
Tardive dyskinesia
Acetyl choline
Histamine
Melatonin
Аn individual is admitted to the emergency room of a
hospital after taking a drug of abuse that destroyed
selective groups of neurons in the brainstem After the
individual became ambulatory, he became chronically
depressed Which of the following neuronal groups in the
brainstem might be related, either directly or indirectly,
to this person's condition?
In the experiment, an animal had its brain stem cut,
which caused a rapid increase of extensor muscle tone
(decerebrate rigidity). This condition arose because the
muscles were no more under the control of the following
brain structure:
A patient complains of rapid fatigability. Objectively: he
staggers and overbalances in the upright position with
closed eyes. Skeleton muscular tonus is decreased. What
brain structure is most likely to be damaged?
An individual is admitted to the emergency room of a
hospital after taking a drug of abuse that destroyed
selective groups of neurons in the brainstem After the
individual became ambulatory, he became chronically
depressed The neurotransmitter loss most likely linked
to the symptoms is which of the following?
An individual is admitted to the emergency room of a
hospital after taking a drug of abuse that destroyed
selective groups of neurons in the brainstem After the
individual became ambulatory, he became chronically
depressed An accepted approach toward the treatment of
this disorder would be to administer which of the
following?
The temperature regulating centre is mainly located in:
A patient shows involuntary athetoid movements, he
probably has:
The patient with disorientation and memory loss died An
autopsy was taken of his brain and regional brain
chemistry and neuropathology identified Which of the
following is the likely sites where neuropathology was
identified?
Raphe nuclei
Vestibular nuclei
Nucleus ambiguus
Trigeminal
nucleus
Red nucleus
Blue spot
Black substance
Striatum
Cerebellum
Thalamus
Hypothalamus
Precentral gyrus of Basal ganglions
cerebrum cortex
Serotonin
Enkephalin
Dopamine
Norepinephrine
Gamma
amino
butyricacid
(GABA)
spinal Dorsal
column
nuclei
Gray tuber
Glycine
A
serotonin A CNS depressant
reuptake inhibitor
(SSRI)
A
dopaminergic A
noradrenergic An NMDA blocker
antagonist
antagonist
Hypothalamus
Lesions in basal
ganglion
Cerebral
cortex,
basal nucleus of
Meynert,
hippocampus
Pons
Hypothalamic
lesions
Substantia nigra,
midbrain
periaqueductal
gray, ventrolateral
thalamus
Medulla
Lesions in the
cerebral cortex
Cerebellar cortex,
hypothalamus, red
nucleus
Cerebral cortex
Cerebellar lesions
Cerebellum
Thalamic lesions
Nucleus gracilis, Fastigial nucleus,
deep
pontine subthalamic
nuclei, vestibular nucleus, superior
nuclei
colliculus
222.
2
223.
2
224.
2
225.
2
226.
2
227.
2
228.
2
229.
2
230.
2
231.
2
232.
2
233.
2
234.
2
235.
2
236.
2
237.
2
Which of the following statements about the precentral
gyrus is true?
Hypotonia, as seen in spasticity, rigidity, spasm, and
cramps, usually occurs with disorders of the:
Which of the following statements about the basal nuclei
is true?
It is involved in It is involved in
motor control
sensory perception
Cerebellum
Descending
pathways
They contain the They are located in
caudate nucleus
the cerebellum
It is located in the
temporal lobe
Ascending
pathways
They are involved
in sensory control
Involved in visual
perception
Medulla oblongata
and pons
They are part of the
pyramidal system
Involved
in
auditory perception
Precentral gyrus
Their lession result
in
decerebrate
rigidity
Midbrain
Which of the following acts as a center for coordination The cerebellum
movements?
The reticular activating system:
Is
described
correctly by all of
the above
The thalamus
The hypothalamus
The red nucleus
Is composed of
neurons that are
part of the reticular
formation
In decerebrate animal following reflexes will be present Righting reflex
exept:
Cerebellectomy in the primate results in:
Ataxia
Stretch reflex
Disturbed vision
Is
a
loose
arrangement
of
neurons with many
interconnecting
synapses
Crossed extensor
reflex
Hypertonia
All of the following symptoms pertain to cerebellar Tremor at rest
lesions except one Identify the exception
In the control of emotion and motivation, the limbic The hypothalamus
system works together with:
Verbal ability predominates in:
The
left
hemisphere
of
right-handed
people
The consolidation of short-term memory into long-term The hippocampus
memory appears to be a function of:
The superior bodies in the corpora quadrigemina of the Visual reflexes
midbrain are involved in:
Intention tremor
Hypotonus
Is located in the Functions to arouse
brain stem and the cerebral cortex
midbrain
to
incoming
sensory
information
Tonic neck reflex
Positive supporting
reaction
Decerebrate
Decorticate rigidity
rigidity
Asynergy
Adiodochokinesia
The pons
The thalamus
The cerebellum
The
right The
right Precentral gyrus
hemisphere of most hemisphere of 97%
left-handed people of all people
The
substantia The
cerebral
nigra
peduncles
Auditory reflexes
Relaying
of
cutaneous
information
Medulla oblongata Cerebellum
Highly skilled movements that become almost automatic Midbrain
are controlled by:
Comprehension poor; speech is fluent are involved in:
Damage
to Damage to Broca's
Wernicke's area
area
In most people, the right hemisphere controls movement The left side of the The head and neck
of:
body primarily
only
The inferior bodies in the corpora quadrigemina of the Auditory reflexes
midbrain are involved in:
Relaying
cutaneous
information
Damage to angular
gyrus
The right side of
the body primarily
The basal nuclei
Postcentral gyrus
The
arcuate The
precentral
fasciculus
gyrus
Release of pituitary All of the above
hormones
Thalamus
Damage
to
precentral gyrus
Both the right and
left sides of the
body equally
of Release of pituitary Visual reflexes
hormones
Hypothalamus
Damage
hyppocampus
The head only
All of the above
to
238.
2
239.
2
240.
2
241.
2
242.
2
243.
2
244.
2
245.
2
246.
2
247.
2
248.
2
249.
2
250.
2
251.
2
The cerebellum is important in controlling:
Muscular
coordination
Akinesia, muscular rigidity, changes in facial Basal ganglia
expression, unstable posture (signs of Parkinson’s
disease) related with diminished of dopamine in the:
A patient who presents with an intention tremor, "past- Cerebellum
pointing," and a "drunken" gait might be expected to
have a lesion involving which part of the brain:
An abnormal Babinski reflex indicates damage to which Pyramids
of the following?
Which one of the following hypothalamic nuclei is The
anterior
responsible for the detection of the core body hypothalamus
temperature?
The Klüver-Bucy syndrome is characterized by Amygdala
decreased emotional expression, loss of fear, excessive
oral behavior, and increase in sexual activity These
characteristics of the Klüver-Bucy syndrome are
produced by bilateral lesions of which of the following?
An impairment in the ability to perform certain types of Premotor cortex
learned, complex movements (referred to as apraxia)
usually results from a lesion of which of the following?
Which of the following is the primary transmitter Gaba
released from terminals of both neostriatal and
paleostriatal neurons?
A man has ataxia, unsteady in Romberg test and displays Flocculonodular
a tendency to fall backward or to either side in a drunken lobe
of
the
manner A lesion is most likely located in which of the cerebellum
following?
Which one of the following thalamic nuclei makes local Centromedian
connections with other thalamic nuclei and, additionally, thalamic nucleus
projects to the basal ganglia:
The supraoptic nucleus is most closely associated with Water balance
which of the following?
Lesions of the lateral hypothalamus will likely produce Aphagia
which of the following:
A number of investigations have provided strong Circadian rhythms
evidence that the suprachiasmatic nucleus plays an
important role in which of the following:
Damage to which of the following CNS structures Medullary
caused the difficulty breathing?
respiratory center
Stereognosis
Muscle strength
Stretch reflexes
Posture
Medulla oblongata
Midbrain
Cerebellum
Hypothalamus
Medulla
Cortical motor strip Basal ganglia
Eighth
nerve
Spinal cord
Brainstem
Basal ganglia
The
lateral The
hypothalamus
nucleus
Cerebellum
arcuate The
nucleus
cranial
posterior The paraventricular
nucleus
Hippocampus
Ventral
hypothalamus
Corpus callosum
Cingulate gyrus
Precentral gyrus
Postcentral gyrus
Prefrontal cortex
Cingulate gyrus
Glycine
Enkephalin
Dopamine
Glutamate
Hemispheres of the Anterior limb of Dentate nucleus
posterior cerebellar the internal capsule
lobe
Anterior lobe of the
cerebellum
Medial geniculate Lateral geniculate Dorsomedial
thalamic nucleus
thalamic nucleus
thalamic nucleus
Anterior
nucleus
Feeding behavior
Feeding behaviors
Temperature
Sexual behavior
regulation
Drinking behaviors Sexual behaviors
Short-term memory
functions
Hypertension
Water intake
Food intake
Hypertension
Short-term memory
Diencephalon
Pons
Phrenic
innervating
diaphragm
thalamic
nerve Trigeminal nerve
the
252.
2
253.
2
254.
2
255.
2
256.
2
257.
2
258.
2
259.
2
260.
2
A patient experiences difficulty in walking down stairs
and reports some double vision as well Which of the
following is the most likely locus of the lesion?
Deviation of the tongue to the left, away from the right
hemiparesis, implies a lesion in which area of the
nervous system?
Which two nuclei mediating sensation of the arms, face,
legs, and trunk may have sustained damage from the
original stroke?
Midbrain
Medulla
Dorsal pons
Ventromedial pons
Spinal cord
Left hypoglossal Right hypoglossal Right
inferior Left inferior frontal Right
nucleus
nucleus
frontal lobe
lobe
peduncle
Ventral posterior
lateral and ventral
posterior
medial
nuclei
of
the
thalamus
When a patient is asked to follow an object when it is Midbrain
placed in the right side of his visual field, he is unable to
move his right eye either up or down The lesion is most
likely situated in which of the following:
A patient is capable of displaying pupillary constriction Pretectal area
during an accommodation reaction but not in response to
a direct-light stimulus The lesion is most likely present
in which of the following:
A patient delays initiation of movement, displays an Hemispheres of the
uneven trajectory in moving her hand from above her posterior cerebellar
head to touch her nose, and is uneven in her attempts to lobe
demonstrate rapid alternation of pronating and
supernating movements of the hand and forearm Which
of the following is the most likely locus of the lesion:
After car accident and multi traumatic injury in patient Midbrain
developed chronic pain and he was subsequently treated periaqueductal gray
with narcotic Consequently, he developed an addiction
to these drugs In which a region of the brain has this
receptor been shown through extensive research to be
heavily concentrated?
Patient neurologic exam was normal, but she had the Adenohypophysis
headaches, visual problem and high prolactin levels and
revealed changes on magnetic resonance imaging test of
her head A tumor in which of the following areas could
cause a high prolactin level?
Patient facial expression was masklike, with few Substantia nigra,
eyeblinks When him asked to write a sentence, the pars compacta
letters became progressively smaller toward the end of
the sentence His speech was soft and monotonous, and
he had a slow, resting pill-rolling tremor in both of his
hands He had very little spontaneous movement, and his
Lateral and medial Putamen
and Caudate
geniculate nuclei of globus pallidus
putamen
the thalamus
cerebral
and Anterior and lateral
dorsal nuclei of the
thalamus
Medulla
Basilar aspect of Pontine tegmentum Cerebellum
the pons
Optic nerve
Ventral
cell Visual cortex
column of cranial
nerve III
Flocculonodular
Vermal region of Fastigial nucleus
lobe
of
the the
anterior
cerebellum
cerebellar lobe
Edinger-Westphal
nucleus of cranial
nerve III
Ventral
spinocerebellar
tract
Mammillary bodies Precentral gyrus
Inferior
nucleus
olivary Deep
nuclei
pontine
Amygdala
Hippocampus
Adrenal glan
Neurohypophysis
Caudate nucleus
Thalamus
Red nuclei
Substantia
gelatinosa
261.
2
262.
2
263.
2
264.
2
265.
2
266.
2
267.
3
268.
3
269.
3
270.
3
271.
3
272.
3
273.
3
274.
3
arms, legs, and trunk were stiff Damage to which of the
following structures in particular causes these problems:
The superior bodies in the corpora quadrigemina of the Visual reflexes
midbrain are involved in:
Highly skilled movements that become almost automatic Midbrain
are controlled by:
The inferior bodies in the corpora quadrigemina of the Auditory reflexes
midbrain are involved in:
The cerebellum is important in controlling:
Muscular
coordination
Akinesia, muscular rigidity, change in facial expression, Basal ganglia
unstable posture (signs of Parkinson’s disease) related
with diminished of dopamine in the:
Motor aphasias is result of damage of area, which is Broca’s area
located in:
The division of the ANS that arises from the thoracic Sympathetic
and lumbar regions of the spinal cord is:
The division of the ANS that arises from the brainstem Parasympathetic
and sacral region of the spinal cord is:
Stimulation of β-adrenoreceptors will cause contraction Bronchioles
of the smooth muscle in the:
Norepinephrine will cause contration of the smooth Arterioles
muscels in the:
The interstitial cells of Cajal have all these properties Triggers
gut
except one Identify the exception
secretion
Auditory reflexes
Medulla oblongata
Relaying
cutaneous
information
Stereognosis
Relaying
cutaneous
information
Cerebellum
of Release of pituitary Motor control
hormones
of Release of pituitary Visual reflexes
hormones
somatic
special
afferent,
visceral
Hypothalamus
Motor control
Muscle strength
Stretch reflexes
Posture
Medulla oblongata
Midbrain
Cerebellum
Hypothalamus
Wernicke’s area
Angular qurus
Hippocampus
Parasympathetic
Pupils
Sympathetic and Enteric
parasympathetic
Sympathetic and Enteric
parasympathetic
Intestine
Arterioles
Amygdaloid
nucleus
Sympathetic
enteric
Sympathetic
enteric
Ciliary body
Bronchioles
Pupils
Ciliary body
Sympathetic
Serve as electrical Fire
action
pacemakers
potentials
to
initiate
the
electrical
slow
waves propagating
along the GI tract
Which of the following is the origin of electrical slow The
interstitial The smooth muscle The smooth muscle
wave activity in gastrointestinal tract smooth muscle:
cells of Cajal
of the circular of the longitudinal
muscle layer
muscle layer
Dorsal motor nucleus of the vagus :
Superior salivatory Nucleus ambiguus Inferior salivatory
nucleus
nucleus
The vagus nerve (cranial nerve X) includes which of the General
following components:
afferent,
visceral
general
Thalamus
Special
afferent,
sensory
general
visceral
special
afferent,
visceral
Intestine
+
+
Are found between Hormonally
the
longitudinal regulate peristalsis
and
circular
smooth
muscle
layers
The smooth muscle
of the muscularis
mucosa
Edinger-Westphal
nucleus of cranial
nerve III
General
visceral General
visceral
afferent
and efferent and special
general
visceral visceral
efferent
efferent only
only
The
plexus
myenteric
Red nucleus
Special
efferent,
visceral
and
visceral
general
efferent,
general
275.
3
276.
3
277.
3
278.
3
279.
3
280.
3
281.
3
282.
3
283.
3
284.
3
afferent,
and
general
visceral
efferent
Arteriovenous anastomoses in the skin:
Close in response
to
sympathetic
nerve activity
Neural regulation of vascular resistance in skeletal Sympathetic nerves
muscle is mediated by:
alone
afferent,
and
general
visceral
efferent
Show basal tone Are responsive to
and autoregulation metabolic activity
of the skin
Parasympathetic
Motor nerves
nerves alone
Brain-gut axis is:
Cooperation
between
sympathetic
and
parasympathetic
parts of ANS
Parasympathetic
Cooperation
between
enteric
and
parasympathetic
parts of ANS
Mydriasis or pupillary dilatation is response to Sympathetic
activation:
Mydriasis or pupillary dilatation, involves all the Contraction
of
following except one Identify the exception
radial fibers of the
iris
The neurologist noted that the patient right pupil was
smaller than the left; it was still reactive to light Also his
right eyelid drooped slightly These signs are due to
which of the following:
In patient was diagnosed the absence of active
sympathetic innervation of the eye, orbit and face, the
pupil constricts and the upper eyelid droops partly
(Horner's syndrome) Which pair of neurotransmitters is
involved in the pathway that has been damaged?
Which of the following nuclei gives rise to
parasympathetic preganglionic axons that travel in the
glossopharyngeal nerve?
Which of the following statements about the enteric
nervous system is correct?
Which of the following statements is incorrect?
Cooperation
between
sympathetic
and
enteric parts of
ANS
Sympathetic
+
parasympathetic
Relaxation of the Stimulation of the
sphincter muscles Edinger-Westphal
of the iris
nucleus
visceral
only
Are insensitive to
circulating
epinephrine
Combined action
of sympathetic and
parasympatetic
nerves
Long and short
vago-vagal reflexes
afferent
Show
reactive
hyperemia
Sympathetic
cholinergic nerves
Cooperation
between
sympathetic
CNS
and
Sympathetic
+ Enteric
enteric
Sympathetic nerve Impulse
discharge
transmission by the
superior cervical
ganglion
Sympathetic nerves Parasympathetic
Combined action Enteric
nervous Sympathetic
alone
nerves alone
of sympathetic and system
cholinergic nerves
parasympatetic
nerves
Ach
and Substance P and Norepinephrine
5-HT and GABA
GABA and ach
norepinephrine
acetylcholine (ach) and epinephrine
Inferior salivatory Dorsal
nucleus
nucleus
motor Edinger-Westphal
nucleus
The
myenteric The
enteric
plexus
regulates nervous system is
fluid homeostasis
made up of only a
few
hundred
neurons
The gall bladder The gall bladder
concentrates
the has
bile constituents by parasympathetic
sympathetic
influence
There are primary
afferent neurons in
the
myenteric
plexus
Nucleus
ambiguous
The
submucosal
plexus coordinates
the contractions of
the
gut
wall
muscles
Gastric
juice Normally
volume is increase pancreatic juice is
by parasympathetic stimulate
by
influence
parasympathetic
Superior salivatory
nucleus
Intestinal
contractions cease
when the gut is
removed from the
body
Salivation
has
synergetic
influence by both
part of ANS
285.
3
286.
3
287.
3
288.
3
289.
3
290.
3
291.
3
292.
3
293.
3
294.
3
295.
3
296.
3
influence
Which receptor antagonist could be used to block fast Curare
excitatory transmission in a sympathetic ganglion?
Which of the following characteristics is a feature of Decrease heart rate
Dannini-Ashnera's syndrome?
after
eyeball’s
stimulation
The specific effect of sympathetic stimulation on Increase
metabolic rate is:
What the followings are correct in relation to control of Chewing food can
gastric juice formation:
reflexly stimulate
nvagus to initiate
secretion
Atropine
influence
Prazosin
Bicuculline
Strychnine
Increased sweating Partial ptosis
on the face
Pupillary dilation
Vasoconstriction of
facial skin
Non influence
Inhibition
Depends on basal
metabolic rate
Sympathetic part of
ANS
reflexly
stimulates enteric
ANS to increase
secretion
Contain
actin,
myosin filaments
running parallel to
each other under
paracrine control
Is
effective
increase
by
influence of enteric
ANS
Raise
blood
pressure
Cotrol arterioles
Parasympathetic
influence
is
decreased
in
response to fat, egg
yolk and meat
What the followings is correct in relation GI smooth Contain
Are each directly
muscle cells:
neuromuscular
innervated only by
junctions
with enteric ANS
numerous
neurotransmitters
What the followings is correct in relation heart rate:
Decrease
by Is abolished by
activation
vagotomy
parasympathetic
part ANS
Stimulation of the central (proximal) end of a cut vagus Cause apnea
Increase heart rate
nerve would be expected to:
Sympathetic vasodilator fibres are:
Distributed
to Believed to be
skeletal muscles
cholinergic
in
nature
Patients who are extremely allergic to bee stings will The contraction of The strength of
self-administer epinephrine when stung The activation airway
smooth ventricular muscle
of adrenergic beta-receptors by the injected epinephrine muscle
contraction
will decrease which of the following:
A research physiologist is performing an experiment in Diaphoresis
Bradycardia
which he stimulates sympathetic cholinergic neurons (increased
Which of the following responses is expected?
secretion of sweat
glands)
Which one of the following is the putative inhibitory Vasoactive
Dopamine
neurotransmitter responsible for relaxation of intestinal peptide
gastrointestinal smooth muscle:
A research physiologist is performing an experiment in Diaphoresis
Bradycardia
which he stimulates sympathetic cholinergic neurons (excess
of
Which of the following responses is expected?
sweeting)
Decrease
Sympathetic part is Vagotomia causes
decreased
in acceleration
of
response to fat, egg secretion
yolk and meat
Are
functionally
coupled
with
sympathetic
and
parasympathetic
parts of ANS
Is
under
sympathetic
control
Stimulate
inspiration
Important
temperature
regulation
The
rate
depolarization
the SA node
Has
the
same
innervations that
skeletal
muscle
cells
Decrease
by
activation
sympathetic
part
ANS
Inhibit coughing
in Controlled
by
cerebral cortex
of The transport of
in calcium
into
skeletal
muscle
fibers
Bronchoconstrictio Increased
n
gastrointestinal
motility
The
rate
glycogenolysis
the liver
Somatostatin
Adrenalin
Acetylcholine
Bronchoconstrictio Increased
n
gastrointestinal
motility
of
in
Increased
peripheral vascular
resistance
Increased
peripheral vascular
resistance
297.
3
Nicotinic receptors are responsible for which of the Producing
the Decreasing the rate Increasing
the Delaying
the
following:
skeletal
muscle of
phase-4 force of stomach emptying of liquids
end-plate potential depolarization at contractions
from the stomach
the SA node
298.
3
Chemical blocks alpha adrenoreceptors would cause:
299.
3
300.
3
301.
3
The “homeostatic and rest” division of the autonomic
nervous system is called the:
The “adaptation and fight” division of the autonomic
nervous system is called the:
The following are true about the autonomic nervous
system:
302.
3
Ablation of the stellate ganglion causes:
303.
3
304.
3
305.
3
306.
3
307.
3
The following signs occur in ipsilateral interruption of
the cervical sympathetic trunk, except one Identify the
exception
The effect of sympathetic nervous system include all Constricts
following statements except one Identify the exception
bronchiole smooth
muscle
Increased activity of the sympathetic nervous system Bronchiolar
cause:
dilation
The following are true about the muscarinic receptors M2
muscarinic
except one Identify the exception
receptors mediate a
positive
chronotropic effect
The following are true about the autonomic nervous All
sympathetic
system except one Identify the exception
postganglionic
neurons
are
adrengic neurons
A reduction in Bronchoconstrictio A fall in blood
sweat production
n
pressure due to
slowing of heart
rate
Parasympathetic
Sympathetic
Enteric division
division
division
Sympathetic
Parasympathetic
Enteric division
division
division
The
sympathetic The postganglionic All postganglionic
preganglionic
neurones
are neurones
are
neurones leave the largely myelinated cholinergic
spinal cord via the
dorsal roots of
spinal nerves T1L3
Vasodilatation of Dilatation of the Posteral
the ipsilateral arm ipsilateral pupil
hypotension
Decreased
Small pupil
Enophthalmos
lacrimation
Contraction of the Pupillary dilatation
bladder
detrusor
muscle
Penile erection
Pupillary
constriction
They are found at They
can
be
the postganglionic selectively blocked
parasympathetic
by atropine
synapses
The preganglionic The postganglionic
fibres are mainly fibres are mainly
myelinated, slow unmyelinated
C
conducting B fibres fibres
Constriction
pupil
Decreasing
the
excitability
of
sympathetic
postganglionic
neurons
of Constriction
of
cilliar muscles
Paraventricular
nucleus
Paraventricular
nucleus
The preganglionic
neurones of the
parasympathetic
nervous system are
shorter than the
sympathetic
nervous system
Loss of consensual
light reflex
Ptosis
Limbic division
Reduced
gastrointestinal
motility
Accommodation
for near vision
M1
muscarinic
receptors are found
in the brain
Constricts skin and
mucous membrane
blood vessels
Gallbladder
emptying
M4
muscarinic
receptors are found
in the pancreas
All preganglionic
neurones
are
cholinergic
neurons
The postganglionic
neurons of the
sympathetic
pathway is longer
than that of the
parasympathetic
pathway
Limbic division
The
sympathetic
outflow is only
found in the cranial
nerves
Loss of ipsilateral
lacrimation
Vasodilatation in
the skin of the face
308.
3
Selective vagotomy would probably cause
Bronchodilatation
309.
3
310.
3
Bladder functions are regulated by which of the Lumbar,
sacral,
following combinations of inputs:
and
descending
fibers from the
cerebral cortex
Which of the following gastrointestinal motor activities Orad
stomach
is most affected by vagotomy:
accommodation
311.
3
312.
3
313.
3
314.
4
315.
4
316.
4
317.
4
318.
4
319.
4
320.
321.
4
4
Insulin secretion
Vagal and sacral
efferent fibers only
Secondary
esophageal
peristalsis
The sympathetic response in a "fight or flight" reaction The resistance of The arterial blood
causes a decrease in which of the following?
the airways
pressure
A 55-year-old female patient is admitted to the hospital 2-adrenergic
Muscarinic
ach
for treatment of arterial hypertension The treatment of receptors
receptors
this combination of symptoms by the appropriate drug is
mediated by its actions on which of the following
receptors?
Physiologist noticed that dog right eyelid drooped Sympathetic fibers Left
oculomotor
slightly and that his right pupil was smaller than the left coursing from the nerve
Where in the nervous system has damage occurred?
hypothalamus
to
the
intermediolateral
cell column
Coordination via the endocrine system tends to differ Does not require Has
short-term
from that produced by the nervous system because the conscious activity effects
endocrine system:
Which of the following statements is true of negative Negative feedback Negative feedback
feedback?
is important for makes
any
maintaining
deviation from a
homeostasis in the normal value larger
body
A variable changes in one direction and the system Positive feedback
Negative feedback
responds by changing in the same direction This type of
control is said to be
In negative feedback mechanisms, if a sensor detects an Decreased
Unchanged
increase in a variable, the action of an effector is
A good example of a positive feedback mechanism Enhancement
of Body temperature
would be:
labor contraction
regulation
Control mechanisms that lower blood pressure in Negative feedbacks Positive feedbacks
response to a rapid rise in blood pressure are called:
The hypothalamus inhibits the release of:
GH
TSH
Paracrine communication refers to interactions between Release
and Transmission
of
Gastric
acid
secretion
Vagal, sacral, and
descending fibers
from the cerebral
cortex
Distention-induced
intestinal
segmentation
The diameter of the
pupil
Dopaminergic
receptors
Right
nerve
Salivatory
Bile production
secretion
Lumbar and sacral Lumbar, thoracic,
efferent fibers only and cervical fibers
only
Caudad
stomach Migrating
peristalsis
complexes
motor
The blood glucose The heart rate
concentration
Serotonergic
Gaba receptors
receptors
oculomotor Edinger-Westphal
nucleus
Is faster and more Require
nerve
coordinated
impulse
as
messenger
Negative feedback Negative feedback
occurs when the is a very unusual
uterus
contracts control mechanism
during birth
in the human body
Intermittent
feedback
Cyclic feedback
Increased
Amplified
Parasympathetic
fibers
coursing
from the EdingerWestphal nucleus
Require nerve fibre
as conductor
Negative feedback
is not important for
maintaining
homeostasis in the
body
Reverberating
feedback
Modulated
Regulating glucose Blood
calcium Blood sodium level
levels in the blood level regulation
regulation
Equilibrium
Homeostasis
Dynamic balance
ACTH
Release
FSH
of Release
Oxytocin
of Transmission
of
322.
4
323.
4
324.
4
325.
4
326.
4
327.
4
328.
4
329.
4
330.
331.
4
4
332.
4
333.
4
334.
4
335.
4
cells resulting from direct contact of cells at tight diffusion
of
junctions
chemical mediators
through
extracellular fluid
to target cells
The supraoptic nucleus of the hypothalamus is believed Antidiuretic
to control secretion of which of the following hormones? hormone (arginine
vasopressin)
Which of the following hormones uses cAMP as a Epinephrine
second messenger?
Which of the following terms best describes the Permissive
interactions of insulin and glucagons/
Which of the following correctly describes the role of It stimulates the
inositol triphosphate in hormone action?
release of Ca from
the
endoplasmic
reticulum
Which of the following could decrease the sensitivity of The presence of a
a person to stimulation by the gormone?
competitive
antagonist
Which of the following hormones uses inositol Vasopressin
triphosphate as a second messenger?
Which of the following hormones can penetrate the cell Cortisol
membrane?
Which of the following hormones exerts the least effect Vasopressin
on growth?
Regulation by prostaglandins of neighboring is termed:
Paracrine
Where are the posterior pituitary hormones In neurosecretory
manufactured?
cells that originate
in
the
hypothalamus
There are several types of cells in the anterior pituitary Somatotropes
Which ones secrete growth hormone?
Which one of these anterior pituitary hormones shows Adrenocorticotropi
increased secretion in response to stress?
c hormone
mechanical forces chemical mediators chemical mediators
via
extracellular in
localized into blood to act
filaments
synaptic junctions upon
specific
receptors in distant
target tissues
Oxytocin
Growth hormone
Adrenocorticotropi
c hormone
mechanical forces
via
intracellular
fluids
Testosterone
Cortisol
Insulin
Vit D
Synergistic
Antagonistic
Cooperative
Complementary
The thyroid hormones are responsible for regulating Calcitonin
metabolic rate and calcium and phosphate ion
concentration in the blood Which thyroid hormone
lowers calcium and phosphate ion concentration in the
blood?
Hypothyroidism in infants can result in
Cretinism
Follicle-stimulating
hormone
It
activates It activates protein It
opens
Ca It
opens
Na
adenylate cyclase
kinase
channels in the cell channels in the cell
membrane
membrane
A decrease in the
number of target
cells
Testosterone
A decrease in
adenylate cyclase
concentration
Cortisol
An increase in
phosphodiesterase
concentration
Insulin
The presence of a
noncompetitive
antagonist
Vit D
Histamine
Norepinephrine
Vasopressin
Oxytocin
Growth hormone
Testosterone
T4
Insulin
Endocrine
Neurocrine
Autocrine
In the posterior In the anterior In the brain stem
pituitary
pituitary
Exocrine
In the
pituitary
Mammatropes
Corticotropes
Gonadotropes
Follitropes
Prolactin
Luteinizing
hormone
Growth hormone
Thyroidstimulating
hormone
Thyroxine
Thyroidstimulating
hormone
Triiodothyronine
Grave's disease
Myxedema
Hashimoto's
Kushing syndrom
anterior
Parathormone
336.
4
337.
4
338.
4
339.
4
340.
4
341.
4
342.
4
343.
4
344.
345.
346.
4
4
4
347.
4
348.
349.
4
4
350.
4
351.
352.
4
4
353.
4
354.
4
Which hormone is secreted by the adrenal glands Aldosterone;
causing the kidney to conserve sodium and excrete cortex
potassium ions?
Sex hormones are secreted by:
The inner cortex of
the adrenal gland
and the gonads
Which cells of the pancreas secrete insulin, and which is Beta; lowers
the main action of insulin on blood sugar levels?
What seems to be the cause of juvenile-onset or insulin- Immune
cells
dependent diabetes mellitus (IDDM)?
attack the pancreas
that can then no
longer
produce
insulin
The endocrine gland responsible for the body's circadian Pineal
rhythm is the
In the "second messenger" theory, which is the first Hormone
messenger?
Which is not a function of the hypothalamus?
Secrete FSH
Epinephrine;
medulla
Which of these hormones is stored by the posterior
pituitary?
Which hormone directly affects blood pressure?
Which hormone causes the contractions of labor
Which hormone has the greatest influence on the ability
to secrete milk?
Which hormone causes excessive skeletal growth or
gigantism?
Which hormone has the greatest effect on metabolism?
Which is not a hormone secreted by the anterior
pituitary gland?
Which of the following disease caused by hyperfunction
of thyroid gland?
Which of the following hormone affect Ca balance?
The release of epinephrine and norepinephrine from the
adrenal medulla will cause what type of effect in the
body?
Which disorder of the adrenal gland is mostly caused by
autoimmune disease?
Which hormone is released by the kidneys and
stimulates the production of red blood cells?
Adh
Angiotensin
medulla
disease
i; Cortisol; cortex
Epinephrine; cortex
All
endocrine The gonads
tissues of the body
The inner cortex of The outer cortex of
the adrenal gland
the adrenal gland
Delta; raises
Beta; raises
Alpha; lowers
Delta; lowers
The receptors on
the target cells
become no longer
responsive
to
insulin
Thymus
Obesity seems to
be
the
most
common cause of
IDDM
Pituitary
The
individual
consumes
too
much sugar, which
causes an overload
in the bloodstream
Parathyroid
The receptors on
the target cells
become
overresponsive to
insulin
Gonads
Ca
Receptor
Enzyme
Adenylcyclase
Affect heart rate
Fsh
Control
temperature
Lh
Affect
balance
Acth
Adh
Oxytocin
Prolactin
Insulin
Prolactin
Oxytocin
Th
Estrogen
Estrogen
Acth
Progesterone
Progesterone
Testosteron
Growth hormone
Testosteron
Somatotropin
Testosterone
Insulin
Somatostatin
Vasopressin
Thyroxine
Antidiuretic
hormone
Graves' disease
GH
TSH
Follicle-stimulating Gonadotropins
hormone
Myxedema
Thyrotoxic crisis
Acth
Luteinizing
hormone
Diabetes mellitus
Adrenalin
Adrenalin
Calcitonin
An increase
blood pressure
Addison's disease
Erythropoietin
water Control
lipid
balance
Growth hormone
Diabetes insipidus
Glucocorticoids
Mineralocorticoids Cortisone
Testosteron
in An increase in A drop in the heart A drop in blood Fall
in
blood
glycogen storage
rate
glucose levels
pressure
Cushing's
syndrome
Angiotensin II
Myxedema
Graves' disease
Diabetes insipidus
Renin
Natriuretic peptide
Calcitriol
355.
4
356.
4
357.
4
358.
4
359.
4
360.
4
361.
4
362.
4
363.
4
364.
4
365.
4
366.
367.
4
4
368.
369.
4
4
370.
4
371.
4
372.
4
Which of the following is a function of aldosteron?
Regulate
the
absorption
of
sodium
How does insulin act?
Tyrosine
kinase
membrane receptor
Which of the following is the hormone of the pineal Melatonin
gland?
Which of the following hormones has as its target an Tropic hormones
endocrine gland?
Which of the following promotes breast development Prolactin
and stimulates milk secretion?
Which of the following decreases plasma calcium Calcitonin
concentration?
Which of the following hormones increase uterine Oxytocin
contractility?
Which of the following hormones produces Vasopressin
vasoconstriction in arterioles?
Which of the following is a condition resulting from Acromegaly
hypersecretion of growth hormone after adolescence?
Which of the following condition occurs when Gigantism
hypersecretion of growth hormone begins in early
childhood?
Which of the following are released into the blood Neurohormones
specifically by neurosecretory neurons?
Thyroid cells are the only cells in the body to use
Iodine
Vitamin D
Is formed from a
precursor molecule
that is found when
skin is exposed to
ultraviolet light
ADH secretion is decreased by:
Alcohol
Which one of the following hormones is secreted by the Oxytocin
posterior pituitary gland
Inhibition
of Antioxidant
reproductive
activity
function
Voltage gated ion Nuclear receptor
channels
PGH
Melanin
Establish day-night Control of growth
cycles of activity
Paracrines
Renin
Nontropic
hormones
Somatomedins
G protein
cAMP
MSH
Melanocyte
Neuromodulators
Neurotransmitters
Oxytocin
Parathyroid
hormone
Vasopressin
Glucagon
Leutinizing
hormone
Cortisol
PRH
GHRH
Testosteron
Oxytocin
Aldosterone
Cortisol
ACTH
Achondroplastic
dwarfism
Achondroplastic
gigantism
Laron dwarfism
Gigantism
Acromegaly
Laron gigantism
Exophthalmic
goiter
Dwarfism
Hormones
Neurotransmitters
Paracrines
Neuromodulators
Potassium
Stimulates calcium
and
phosphate
uptake in the small
intestine
Iron
Is formed from a
precursor molecule
that is modified in
the
liver
and
kidneys
Nicotine
Growth hormone
(GH)
Sulfur
Stimulate growth
Calcium
Affect na balance
Morphine
Nausea
ThyroidProlactin
stimulating
hormone (TSH)
Which of the following is an indication that ovulation A drop in body An increase in An increase in An increase in
has taken place
temperature
serum
estrogen serum LH levels
serum progesterone
levels
levels
Plasma levels of calcium can be increased most rapidly Intestine
Skeletal
Kidney
Thyroid gland
by the direct action of parathyroid hormone on which of
musculature
the following?
Which one of the following conditions experienced by Increased cardiac Increased incidence Decreased
facial Increased gastric
Oxytocin
Hypoxia
Adrenocorticotropi
c hormone (ACTH)
An increase in
serum FSH levels
Bones
Decreased
pregnant women may be due to the effects of pregnancy
373.
4
374.
4
375.
4
376.
4
377.
4
378.
4
379.
4
380.
4
381.
4
382.
4
383.
4
384.
4
385.
4
386.
4
output
Which one of the following is associated with prolonged Increased urinary
fasting (3 or more days)?
excretion
of
nitrogen
The supraoptic nucleus of the hypothalamus is believed Antidiuretic
to control secretion of which of the following hormones hormone (arginine
vasopressin)
Which of the following is the source of estrogen and Placenta
progesterone during the last seven months of pregnancy
Which of the following is the source of estrogen and Corpus luteum
progesterone during the first two months of pregnancy?
Which of the following hormones is involved in the Oxytocin
ejection of milk from a lactating mammary gland?
Which of the following is an effect of primary Hypertension
hyperaldosteronism (Conn''s syndrome)?
Hyperthyroidism is characterized by which of the
following
A person with hypothyroidism would exhibit which of
the following symptoms?
Biological actions of estrogens include which of the
following?
Increased
basal
metabolic rate
Decreased
metabolik rate
Delayed bone loss
at menopause
of heartburn
acne
Follicle-stimulating Oxytocin
hormone
emptying leading afterload
to
intestinal
cramping
and
diarrhea
Increased glucose Decreased lipolysis
utilization by the
brain
Growth hormone
Adrenocorticotropi
c hormone
Ovary
Corpus luteum
Anterior pituitary
Posterior pituitary
Ovary
Placenta
Anterior pituitary
Posterior pituitary
FSH
LH
Growth hormone
Prolactin
Increased
hematocrit
Decreased
extracellular
volume
Increased secretion Decreased
of insulin
gluconeogenesis
Anorexia
Increased
fluid concentrating
ability
of
kidney
Bradycardia
Decreased
sweating
Tachycardia
Increased
metabolic rate
Inhibition
of Decreased
follicular growth
progesterone
receptors
Which of the following hormones is primarily Follicle-stimulating Chorionic
Estradiol
responsible for development of ovarian follicles prior to hormone
gonadotropin
ovulation
The anti-inflammatory effect of cortisol treatment is Stabilization
of Increased capillary Increased release
thought to be due to which of the following?
cellular lysosomal membrane
of pyrogen from
membranes
permeability
granulocytes
Which of the following gastrointestinal hormones is Motilin
Gastrin
Secretin
involved in initiation of migrating motor complexes?
Which one of the following hormones initiates a Epinephrine
Progesterone
Estrogens
biological effect by activation of cell membrane
receptors
Which one of the following hormones interacts with a Cortisol
Insulin
Thyrotropincytoplasmic receptor, then localizes in the nucleus and
releasing hormone
directs protein and nucleotide synthesis
Hyperkalemia
the
Increased weight
gain
Heat intolerance
Decreased
body
mass index
Increased glucose Decreased serum
tolerance
LDL cholesterol
Progesterone
Luteinizing
hormone
Activation
of Increased
phospholipase A2 formation
leukotrienes
Somatostatin
Enterooxyntin
Cortisol
Thyroxine
Epinephrine
Luteinizing
hormone
of
387.
4
388.
4
389.
4
390.
4
391.
4
392.
4
393.
4
394.
4
395.
4
396.
4
397.
4
398.
4
The signs and symptoms of a patient with primary Hyperkalemia
adrenal insufficiency include which of the following
Thyroid hormones:
Stimulate the rate
of metabolism to
increase
Thyroxine functions to:
Increase the rate at
which energy is
made available to
cells
A 37-year-old patient has has lost 5 kg in weight over Thyroxine
the past three months, he complains of hand tremor,
excessive sweating, exophthalmos, tachycardia. These
changes might have been caused by the increased
secretion of the following hormone:
After a person had drunk 1,5 liters of water, the amount Vasopressin
of urine increased significantly, and its relative density
decreased to 1,001. These changes are a result of
decreased water reabsorption in the distal nephron
portion due to reduced secretion of:
A 32-year-old patient consulted a doctor about the Prolactin
absence of lactation after parturition. Such disorder
might be explained by the deficit of the following
hormone:
ADH would probably cause
Increases
freewater transfer from
the tubular urine to
the capillary blood
Sertoli cells and granulosa cells have which of the They
are
following in common
stimulated
primarily by FSH
Human chorionic gonadotropin is secreted by
The placenta
Pallor
Low ACTH levels
Hypertension
Decrease oxygen
consumption by the
cells
Reduce the rate at
which
carbohydrates are
utilized
Cortisol
Cause a reduction
in heat production
by the cells
Inhibit
activities
within the nervous
system
Stimulate the rate Inhibit growth
of metabolism to
decrease
Inhibit the rate at Inhibit growth
which
calcium
enters body fluids
Insulin
Glucagon
Thyrocalcitonin
Aldosterone
Angiotensin II
Renin
Prostaglandins
Somatotropin
Vasopressin
Thyrocalcitonin
Glucagon
Decreases sodium
resorption from the
ascending limb of
the loop of Henle/
They secrete large
amounts
of
estradiol
Increases plasma Inhibits
osmolality
secretion
The
anterior
pituitary
Steroid hormones are secreted by
The adrenal cortex The
adrenal
and gonads
medulla
The principal action of thyroxin is to:
Increase
Cause
the
catabolism,
and deposition
of
thus, raise energy calcium into bones
availability
All of following factors will increase metabolic rate Removal of the Ingestion of food
except one Identify the exception
thyroid gland
High cortisol levels
ACTH Decreases
reabsorption
urea
They secrete large They
are They are separated
amounts
of stimulated
from germ cells by
testosterone
primarily by LH
a
basement
membrane
The
posterior The thymus
The pineal gland
pituitary
The gonads
The thyroid
The gonads and
thyroid gland
Lower the blood Stimulate
the Rupture
the
sugar level of an thyroid gland to graafian
follicle
individual
secrete tsh
and
cause
ovulation
Exercise
Sympathetic
Increased
body
stimulation
temperature
The alpha cells of the pancreatic islet cells secrete It elevates blood
glucagon What is the function of glucagon?
glucose
by
increasing the rate
of glycogenolysis
in the liver
The regulation of normal blood sugar levels is Hormonal control,
accomplished by:
such as insulin,
glucagon
and
adrenalin
Insulin would probably act to
Antagonizes
the
action of glucagon
399.
4
400.
4
401.
4
402.
4
Parathhormone has a tendensy to cause wich of the Hyperphosphaturia
following conditions
403.
4
A high plasma Ca2+ level causes
404.
4
Renin is secreted by
405.
4
A state of cortisol excess would lead to
406.
4
Which of the following statements about the adrenal It secretes
cortex is true
androgens
407.
4
408.
4
409.
4
Which of the following hormones causes increased atrial
pressure and decreases sodium reabsorption in the
kidneys?
Which of the following is not an anterior pituitary gland
secretion?
Thyroid hormone T3 does not have which of the
following functions?
410.
4
411.
4
Increased
formation of 24,25dihydroxycholecalc
iferol
Juxtaglomerular
cells
Increased secretion
of insulin
some
Atrial natriuretic
peptide
Vasopressin
Like insulin it It lowers
enables the cells to sugar
utilize glucose
blood It
decreases The
lack
of
gluconeogenesis
glucagon
causes
diabetes
Glucagon secretion The lipogenesis
by the pancreas
causing cell tissue
Glycogenesis
lipogenesis
and Glycogenesis
glycogenesis
and
Rise blood sugar Promotes
levels
breakdown
fat Increases
Synergistic
with
breakdown
of adrenalin
glycogen
to
glucose in the liver
Alkalemia
Hypercalciuria
Bone synthesis
Decreases in the
size of the labile
bone pool
Bone
Increased
Decreased
Decreased blood
demineralisation
formation of 1,25- secretion
of coagulability
dihydroxycholecalc calcitonin
iferol
Cells in the macula Cells
in
the Cells in the distal Cells
in
the
densa
proximal tubules
tubules
peritubular
capillary bed
Positive nitrogen Reduction in urea Decreased plasma Increased
bone
balance
synthesis
sodium
formation
concentration
It is innervated by The
zona The
zona The
zona
the nerve fibres
glomerulosa
fasciculata
is fasciculata
is
secretes
stimulated by FSH stimulated by GH
hydrocortizon
PTH
Aldosterone
Vasopressin
Testosterone
TSH
GH
Prolactin
FSH
Decrease calcium Stimulate
bone Create
beta- Cause
brain Increase calcium
reabsorption
development and adrenergic
development
reabsorption
growth
responses
Which of the following does not require the precursor Acth
Cortisol
Testosterone
Aldosterone
Hydrocortisole
pregnenolon?
Which of the following is not a function of Causes activation Causes increased Causes
some Causes increased Control Na balance
progesterone?
of FSH
body temperature
smooth
muscle spiral artery growth
Which of the following is not a function of estrogen?
Decreased overall
transport proteins
Most of the testosterone secreted by the testes exists in Testosterone bound
the plasma in the form of
to
sex-steroidbinding globulin
412.
4
413.
4
414.
4
Which of the following GI hormones is most important Cholecystokinin
in initiating gall bladder contraction?
(CCK)
415.
4
416.
4
Which of the following areas of the adrenal gland would
you expect to increase in activity in a patient subjected
to salt restriction?
Having to adjust to a dark room after walking in from
the sunshine is explained by the fact that:
417.
5
418.
5
Color vision:
419.
5
420.
5
421.
5
422.
423.
5
5
Impulses originating in the organ of Corti in the cochlea
are carried chiefly to what part of the cerebrum?
The area of most acute and detailed vision, where the
cones are concentrated for daytime visualization, is the
The rapid control of equilibrium(as when running) is
maintained primarily as a result of the sensory
information coming from :
Which of the following is nessessary to hear sounds?
Organ of Corti
Which of the following statements is/are correct for the B and D only
rods of the retina?
Zona glomerulosa
of the adrenal
cortex
Rods exposed to
strong light need
time to regenerate
rhodopsin so that it
can
again
be
broken down to
produce
nerve
impulses
Which of the following has less refractory properties of Curvature of the
the eye
retina
Causes
breast
growth
Dihydrotestosteron
e bound to gonadal
steroid-binding
hormone
Gastric inhibitory
peptide (GIP)
Adrenal medulla
relaxation
Causes inhibition Increased follicle
of FSH
development
Free
Free testosterone
dihydrotestosterone
Gastrin
Decreased follicle
development
Testosterone bound
to albumin
Secretin
Vasoactive
intestinal
polypeptide (VIP)
Zona fasciculata of Zona reticularis of Zona fasciculata +
the adrenal cortex the adrenal cortex zona reticularis
Rhodopsin
does The breakdown of Only cones can Cones and rods can
not function in dim rhodopsin
to function in dim function in dim
light
scotopsin
occurs light
light
slowly
The cornea
The
anterior The
posterior The
anterior
surface of the lens surface of the lens surface of the lens
+ the cornea
Is a function of Is most acute in Depends
on Depends on the Depends
on
cone cells
dim light
different
pigments known as different
wavelengths
of rodopsins
wavelengths
of
colored
light
colored
light
stimulating
the
stimulating
the
rods
cones
Temporal lobe
Prefrontal lobe
Posterior part of Parietal lobe
Post central gyrus
the frontal lobe
Macula lutea
Optic papilla
Ciliary body
Suspensory
Stratum corneum
ligaments
The
vestibular The proprioceptors The eyes
The proprioceptors The ears
apparatus
in the neck
in the body
Chorda tympany
Uses rhodopsin as
its light-sensitive
chemical
Semicircular canals
Requires vitamin D
for the synthesis of
photo chemicals
Vestibulum
Cerebellum
Are thought to be Does not involved
functional
in in perception of
adaptation
to colors or daytime
424.
5
The middle ear:
Transmits
and
amplifies
sound
waves to the inner
ear
Causes the circular
muscles of the iris
to contract reflexly
on
sudden
exposure to bright
light, thus testing
for the efficiency
of
the
parasympathetic
nervous system
Taste buds at the
sides of papillae of
the tongue
425.
5
The photo pupil or pupillary reflex:
426.
5
Specialized structures for the reception of taste are:
427.
5
Which one of the following statements is not true of the It contains the
fovea centralis?
greatest amount of
rods
428.
5
In the accommodation for close up vision, what The ciliary muscles
adjustments are made?
contract, the lens
becomes
more
convex and the
sphincter of the
pupil constricts
429.
5
For distant vision:
430.
5
431.
5
432.
433.
5
5
The ciliary muscles
are relaxed
When an object is closer than 20 ft,
The
lens
accommodates
The correct pathway for impulses leaving the retina is:
Photo
receptors,
bipolar
cells,
ganglion
cells,
optic nerve
The area that can be seen with the eyes open is the:
Visual field
Action potentials are stimulated in the cochlear neurons Bending the hairs
by
of the hair cells
Is
primarily Is filled with fluid
concerned
with
equilibrium
darkness
Contains
vestibule
vision
the Contains the organ
of corti
Causes the eyelids
to close to prevent
any more light
from entering the
lens
and
thus
injuring
the
vitreous humor and
retina
Are thought to be Controled
by Controled by pons
functional
in medulla oblongata
adaptation
to
darkness
The
glossopharyngeal
and
hypoglossal
nerves
It is a small
depression at the
center
of
the
macula lutea
The muscles of the
ciliary body reflex,
the lens becomes
less convex and the
sphincter of the
pupil relaxes
Located in the Sensitive to
tongue and nasal chemical
mucous membrane substances
The
highest
concentration
of
cones are found in
the fovea centralis
The ciliary muscles
contract, lightening
the
suspensory
ligaments, the lens
flattens, and the
pupil becomes dark
adapted
The ciliary muscles The lens is less flat
are contracted
The ciliary muscles The lens becomes
relax
more concave
Photo
receptors, Photo
receptors,
ganglion
cells, bipolar cells, optic
bipolar cells, optic nerve,
ganglion
nerve
cells
Optic foramen
Optic chiasma
Vibrations of the Vibrations of the
basilar membrane
tectorial membrane
all Described by none
of the above
It is the area of
greatest
visual
acuity and color
discrimination
The ciliary muscles
contract
It contains the less
amount of rods
The lens is more
spherical
The lens becomes
less convex
Photo
receptors,
optic
nerve,
ganglion
cells,
bipolar cells
Optic tract
Vibrating the oval
window
The ciliary muscles
are constricted
The ciliary muscles
relatively relax
Photo
receptors,
ganglion
cells,
optic nerve, bipolar
cells
Visual radiations
Movement of the
otoliths in the
endolymph
The
muscles
the lens
change,
radial
relax
extrinsic
contract,
does not
but the
muscles
434.
5
Looking at a distant object involves:
435.
5
A nearsighted person:
436.
5
The place where some of the optic nerve fibers cross:
Is
the
chiasma
437.
5
In the brain, the:
Occipital
lobes Parietal
lobes
contain the centers contain the centers
of vision
for vision
438.
5
439.
5
440.
5
441.
442.
5
5
443.
5
444.
5
445.
5
446.
5
Increase in the distance from the eye to the near point of The lens becoming The lens becoming
vision can be due to
more rigid with age more pliable with
age
The route of the ganglionic axons beyond the chiasma is Optic tract
Optic foramen
called the
In course of an experiment thalamocortical tracts of an Olfactory
Auditory
experimental animal were cut through. The animal didn't
lose the following sensations:
The olfactory receptors are examples of:
Chemoreceptors
Proprioreceptors
The scala vestibuli of the inner ear is separated from the Vestibular
Oval window
cochlear duct by the
membrane
The area of the cerebrum that contains cells for Temporal
Frontal
interpretation of sound waves is the lobe
The blind spot of the eye:
Is located just Contains any photo
medial
to
the receptor cells
macula lutea
In the absence of the convergence reflex, the visual Double image
Defocused image
condition would be a:
If a person has some missing or defective cells in the He will not be able His balance will be
middle of the length of tectorial membrane in his inner to hear notes in the affected and his
ear:
middle range of eyes will show
pitch in that ear
nystagmus
447.
5
The function of the eustachian tube is to:
Flattening of the Decrease in the
front of the lens
thickness of the
sclera
May
have
an Can have his vision
eyeball that is corrected
by
abnormally long
biconvex lenses
optic Is the optic lobe
Increase in the
activity of the
lacrimal gland
Has an eyeball that
is abnormally short
Change in the
shape
of
the
eyeball
Sees objects in the
same manner as a
person
suffering
from astigmatism
Lies in front of Lacks a medullary
frontal lobes of the sheath
brain
Temporal
fibers Optic chiasma is
cross at the optic the region in which
chiasma
a single impression
is obtained
The lens becoming The vitreous humor
more convex with becoming thicker
age
with age
Optic chiasma
Visual field
Change in the
shape of the orbital
cavity
Focuses the image
behind the retina
Exteroceptive
Visual
Nociceptive
Mechanoreceptors
Tectorial
membrane
Cerebellar
Thermoreceptors
Basilar membrane
Contact receptors
Nrvus cochlearis
Parietal
Occipital
Is the area where
the cochlear nerve
originates
Highly illuminated
or dim image
He will be totally
deaf in that ear
Is the same as the Is the same as the
optic chiasma
optic tractus
Equalize
air Equalize
blood Magnify sounds
pressure on both pressure in all parts
Contains
only
efferent fibers
Interpretation
of
visual image is a
function of the
optic chiasma
Myopia
Optic nerve
Myopia
Hyperopia
He will not hear
notes from the
middle
range
through
and
including
highpitched ones, with
that ear
Help us maintain
our equilibrium
The effects on his
hearing will be
unnoticeable
Drain off extra
secretions of the
448.
5
449.
5
450.
451.
5
5
452.
5
453.
5
454.
5
455.
5
456.
5
457.
5
458.
5
Hypermetropia or hyperopia is an eye defect that:
sides
of
the
tympanic
membrane
Is corrected by a
convex lens
of the body during
exercise
Is corrected by a Is characterized by
concave lens
loss
of
the
spherical shape of
the lens
Destruction of the left optic tract will lead to:
Loss of the right Complete
Complete
fields of vision in blindness of the blindness of the
both eyes
right eye
left eye
Deficiency of vitamin A in the body may lead to:
Night blindness
Glaucoma
Color blindness
When rays of light come to focus behind the retina and Farsightedness & Nearsightedness & Nearsightedness &
are also irregularly bent by the surface of the lens or astigmatism
astigmatism
glaucoma
cornea, the eye described needs correction for:
Define: myopia
An inability to see An inability to A type of narrowdistant
objects understand
this mindedness
clearly
question
lacrimal gland
Is characterized by Lession of nervus
its development in opticus
elderly people
Loss of lateral
fields of vision in
both eye
Astigmatism
Astigmatism only
Loss of the left
fields of vision in
both eyes
Myopia
Glaucoma only
An inability to
focus
on
near
objects due to
changes in the
retina
The function of the middle ear is:
To amplify sound To transmit sound To contain the ear Place of origin of
vibrations to the vibration, reduce ossicles
nervus cochlearis
inner ear
amplitude, equalize
pressure in the
inner ear
Which of the following receptors is responsible for Ruffini ending
Pacinian corpuscle Merkel's disk
Meissner's
measuring the intensity of a steady pressure on the skin
corpuscle
surface?
If a patient is unable to hear high-frequency sounds, the Oval window
Helicotrema
Stria vascularis
Modiolus
damage to the basilar membrane is closest to which of
the following?
Which of the following is the primary function of the Amplify the sound Filter
high- Enable
the Enhance the ability
bones of the middle ear?
stimulus
frequency sounds direction
of
a to
distinguish
from the sound sound stimulus to different
sound
stimulus
be detected
frequencies
During a voluntary movement, the Golgi tendon organ The
tension The length of the The velocity of the The blood flow to
provides the central nervous system with information developed by the muscle
being movement
the muscle being
about which of the following
muscle
being moved
moved
moved
Which one of the following visual lesions will produce A lesion of the A lesion of the left Section of the left Removal of the
extensive loss of peripheral vision in the left visual right visual cortex lateral geniculate
optic nerve
right lens
fields of both eyes?
An inability to see
what you hear
To contain the ear
muscles
Krause ending
Spiral ganglion
Protect the
from damage
ear
The change in joint
angle produced by
the movement
Section of the left
optic tract
Which one of the following hypothalamic nuclei is
responsible for the detection of the core body
temperature?
In which one of the following sensory systems does
stimulation cause the receptor cell to hyperpolarize?
Which receptor should be activated by the tuning fork?
The
anterior The
lateral The
hypothalamus
hypothalamus
nucleus
459.
5
460.
5
461.
5
462.
5
Damage to which of the following sites could result in Solitary nucleus
the selective loss of taste?
Superior
nucleus
463.
5
Aspartate
464.
5
465.
5
466.
5
467.
5
468.
5
469.
5
470.
5
471.
5
472.
5
473.
6
474.
6
Neurotransmitters implicated in pain modulation, which Opiates
and
may be the targets of pain-alleviating drugs, include serotonin
which of the following
Damage to which structure caused the visual defect?
Left
optic
radiations
The conscious perception of movement is mediated by Joint capsules
which of the following receptors
Which of the following is the principal ascending Lateral lemniscus
auditory pathway of the brainstem?
The output of the retina is mediated by which of the Ganglion cells
following
Which of the following is a direct interneuron linking Bipolar cell
the receptor and ganglion cells
An individual is diagnosed with retinitis pigmentosa, Reduced response
which produces a defective opsin This defect will most to light
likely result in which of the following
The terminals of different classes of primary nociceptive Glutamate
and
afferents have been shown to release which of the substance P
following transmitters onto dorsal horn neurons of the
spinal cord?
Which of the following sensory systems is able to utilize Olfaction
a circuit that bypasses the thalamus for the transmission
of sensory information from the periphery to the cerebral
cortex?
Which of the following types of neurologic visual loss Bitemporal
can cause a loss of peripheral vision?
hemianopsia
The type of inhibition when the conditioned reflex is Transmarginal
inhibited by an excessive increase of the strength of the inhibition
conditioned stimulus is called
Which of the following is not characteristic of Failure to form
conditioned reflexes
conditioned
Vision
Hearing
Pacinian corpuscle
C and A fibers
arcuate The
nucleus
Taste
posterior The paraventricular
nucleus
Touch
Merkel's
tactile Ruffini's corpuscle
corpuscle
olivary Inferior salivatory Spinal nucleus of
nucleus
the
trigeminal
nerve
Glutamate
Epinephrine
Right optic nerve
Left optic nerve
Optic chiasm
Smell
Meissner's
corpuscle
Reticular tegmental
nucleus of the pons
Dopamine
and
norepinephrine
Right
optic
radiations
Pacinian corpuscles
Meissner's
Free nerve endings
corpuscles
Brachium of the Medial lemniscus
superior colliculus
Bipolar cells
Horizontal cells
Merkel's receptors
Rods
Trigeminal
lemniscus
Cones
Horizontal cell
Amacrine cell
Optic nerve cell
Golgi cell
Trapezoid body
Degeneration
of Degeneration
of Loss of central Total loss of vision
area 17 of the cone cells
vision
cerebral cortex
Enkephalins alone Glutamate alone
Substance P alone Enkephalins,
substance P, and
glutamate
Conscious
proprioception
Taste
Vision
Audition
Central scotoma
Superior
quadrantanopsia
Conditioned
inhibitor
Homonymous
hemianopsia
Delayed inhibition
Papilledema
Differentiation
Failure to
conditioned
Reciprocal
inhibition
form Disappearance of Rapid formation of Failure to
the
conditioned conditioned
conditioned
form
reflexes when the reflexes when the reflex if it is not
conditioned
and conditioned
and reinforced
from
unconditioned
unconditioned
time to time
stimulus
is stimuli
are
unpleasant for the separated by more
animal
than 2 minutes
475.
6
476.
6
477.
6
478.
479.
6
6
480.
6
481.
6
482.
483.
6
6
484.
6
485.
486.
6
6
487.
6
488.
6
The consolidation of short-term memory into long-term
memory appears to be a function of
The ability to know that the test you are taking is on
white bond paper, and that you are holding a pencil in
your hand, and that the two are distinctly different is
called:
The ability to solve problems on anatomy and
physiology lecture exams, is called:
The ability to recall thought is the definition of
The ability to retain information from a lecture, to
modify it, and then use it later to answer a test question
is called
In order to reason the answer to this question you have
to gain some knowledge about the cerebral cortex Which
of these answers best tells what you need in order to get
the correct answer?
A student is really “up tight” about all science college
courses He doesn't respect his teachers, he is sarcastic to
his classmates and he works by himself These
characteristics indicate his
Emotional memories is related to the
Comprehension good, can speak and write, but cannot
read ( although can see) It is caused by the:
The ability to determine the meaning of “depolarization”
in a Physiology 233 lecture exam is called:
Experimental neurosis is assotiated whith
Comprehension poor, speech is fluent but meaningless It
is caused by the:
Interhemispheric transfer of learning is closely
assotiated with
Which of the following is most closely concerned with
emotions and emotional expression?
Modality
reflexes when, after
the
conditioned
stimulus, there is a
distracting stimulus
before
the
unconditioned
stimulus
The
substantia The
cerebral The
arcuate The
precentral
nigra
peduncles
fasciculus
gyrus
Association
Clarification
Adaptation
Projection
Reasoning
Learning
Conditioning
Memory
Associative
memory
Perception
Short-term
memory
Sensation
Arousal
Long-term memory Sensory memory
Associative
memory
Modulation
Perception
A good personality Modest emotions
Personality
Perception
Reasoning
Judgements
The hippocampus
reflexes when the
conditioned
stimulus is pleasant
for the animal
Habituation
Perception
Ideas
Perception
Intelligence
Amygdala
Corpus callosum
Damage to angular Damage to broca’s
gyrus
area
Learning
Reasoning
Frontal lobe
Pons
Temporal lobe
Damage
to Damage
to Damage
wernicke’s area
precentral gyrus
hyppocampus
Conditioning
Habituation
Pain
Frontal lobe
Corpus callosum
Damage
to Damage to Broca’s
Wernicke’s area
area
Corpus callosum
Frontal lobe
Pons
Amygdala
Temporal lobe
Damage to angular Damage
to Damage
to
gyrus
precentral gyrus
postcentral gyrus
Pons
Amygdala
Temporal lobe
Hippocampus,
Pyramidal system
amygdaloid
nucleus
and
hypothalamus
Arbor vitae
cerebellum
to
of Pneumotaxic and Basal ganglia and
apneustic centers medulla oblongata
of pons
489.
6
490.
6
491.
492.
6
6
493.
6
494.
495.
6
6
In the control of emotion and motivation, the limbic
system works together with
A strong but unbalanced type of nervous system
coincides with the
Narcolepsy is triggered by abnormalities in
A strong, well-balanced, active type of nervous system
coincides with the
A strong, well-balanced type, with a low mobility of
nervous processes coincides with the
An inability to perform the purposeful act is called
A weak type, with quick fatigability coincides with the
496.
6
Sanguine temperament characterizes
497.
498.
6
6
An inabiility to recognize objects is known as
Phlegmatic temperament characterizes
7
The daily production of hydrogen ion from CO2 is Red blood cell
primarily buffered by which of the following?
hemoglobin
Blood viscosity:
Falls as flow rate
rises
499.
The hypothalamus
The pons
The thalamus
The cerebellum
The basal nuclei
Choleric
temperament
Hypothalamus
Sanguine
temperament
Phlegmatic
temperament
Apraxia
Melancholic
temperament
“Lively”,
active
type with highly
mobile
nervous
processes
Agnosia
“Calm”, inactive or
inert,but
strong
type
Sanguine
temperament
Skeletal muscles
Choleric
temperament
Choleric
temperament
Motor aphasia
Choleric
temperament
Strong, unbalanced
type
Phlegmatic
temperament
Medulla oblongata
Phlegmatic
temperament
Sanguine
temperament
Agnosia
Sanguine
temperament
“Calm”, inactive or
inert type
Melancholic
temperament
Olfactory bulb
Melancholic
temperament
Melancholic
temperament
Agraphia
Phlegmatic
temperament
Weak type
None of the above
Agraphia
Weak type
Akinesia
Mixed type
Plasma proteins
Plasma phosphate
500.
7
501.
7
Maternal physiology changes dramatically in last Increased
total
trimmest of pregnancy. Which one of the following blood volume
changes would be expected:
502.
7
503.
7
Compared with intracellular fluid, extracellular fluid A lower potassium
has.
ion concentration
The oxygen dissociation curve is shifted to the right Polycythaemia
with all following, except:
504.
7
Blood:
Apraxia
Motor aphasia
Strong, unbalanced “Lively”,
active
type
type with highly
mobile
nervous
processes
Extracellular
Red blood cell
bicarbonate
bicarbonate
Is independent of Falls
as
the white cell haematocrit rises
count
Increased
Reduced
conversion
of circulating
glucose
to bicarbonate levels
glycogen
A
greater A higher protein
osmolarity
concentration
Hyperthermia
Respiratory
acidosis
Neocortex
Mixed type
Mixed type
Astereognosia
Mixed type
Mixed type
Is independent of Is independent of
vessel diameter
fibrinogen
concentration
Hypercapnia
Increased
hematocrit
A lower chloride
ion concentration
Sickle
cell
anaemia
A
higher
potassiumion
Increased
concentration of
2,3-DPG in the
erythrocytes
Comprises about Is about 8 to 10 Volume cannot be Normally
has Volume can be
7% of body weight liters
measured
by more than 55% found
by
dilution method
formed elements
difference
after
measuring other
body
water
505.
7
The hematocrit
506.
7
507.
7
508.
7
The viscosity of the blood depends almost entirely on
the concentration of
Which of the following substances in the blood has the
most powerful buffer property:
Total blood in a 70 kg man:
509.
7
The oxygen dissociation curve is shifted to the right by:
510.
7
Erythropoietin is a glycoprotein which:
511.
7
Which of the following would normally be less in the
fetus than in the mother?
512.
7
513.
7
514.
7
515.
7
An increase in the P50 of an oxyhemoglobin curve
would result from a decrease in which of the following?
A 65-year-old female with renal failure presents for
hemodialysis She is found to be anemic and is given a
dose of erythropoietin along with her usual vitamin and
mineral supplements Erythropoietin stimulates which
of the following intermediates in hematopoiesis?
The principle site of production of erythropoietin is
thought to be:
Cyanosis is caused by:
516.
7
517.
7
518.
7
Of
a
normal
person
averages
about 45%
Red blood cells
Of
a
normal
person
averages
about 55%
White blood cells
Is the percent of Is determined by
plasma volume in centrifuging blood
the blood
in a calibrated tube
Thrombocytes
Hemoglobin
Hemoglobin
Bicarbonates
Phosphates
Plasma proteins
Comprises
15%
of
weight
about Comprises
a
body higher percentage
of body weight in
fat people than in
thin people
Increased pCO2
Decreased pCO2
components
Depends on the
altitude at which
the person resides
Plasma
immunoglobulins
Glucose
Volume may be Volume tends to
calculated
if rise when fluid is
haemoglobin
lost
concentration
is
known
Increased pH
Increased
N2
tension
Stimulates red cell Is broken down in Has a half life of Levels inversely
blood production
the kidney
days
proportional
to
haematocrit
PaO2
PaCO2
Pulmonary
Affinity
of
vascular resistance hemoglobin
for
oxygen
pH
Metabolism
Temperature
Oxygen pressure
Comprises about
7% of body weight
Colony forming Basophilic
units-erythroid
erythroblasts
Multipotential
stem cells
Proerythroblasts
Reticulocytes
Kidney
Liver
Spleen
Intestine
Hypoxia
Red bone marrow
Carbon monoxide A
decreased An
increased
poisoning
concentration of concentration of
oxyhaemoglobin
reduced
haemoglobin
The adverse effect of breathing air contaminated with CO
combining CO blocking the The irritant action The direct action
CO is caused by:
with haemoglobin action
of of CO in the of CO upon the
erythropoietin
respiratory
enzymes of the
membrane
cell
Arterial PO2 is reduced in:
Pulmonary
Anaemia
CO poisoning
KCN poisoning
hypoventilation
The following are associated with carbon monoxide It is used for It has nearer 200 It
shifts
the In cigarette, smoke
Decreased
N2
tension
Stimulates red and
white
cell
production
Arterial hydrogen
ion concentration
2,3-DPG
A
decreased
concentration of
haemoglobin
CO
interferring
with the action of
carbonic
anhydrase
During exercise
Can
cause
519.
7
520.
7
521.
7
522.
7
523.
7
524.
7
525.
7
526.
7
527.
7
except:
determining
than 20 times the oxygen
diffusion capacity affinity of oxygen dissociation curve
of the lungs
for haemoglobin
of haemoglobin to
the right
can result in 510%
of
the
smoker's
haemoglobin
being bound to
carbomonoxidaction
than Are
the
less
numerous cells in
the blood
poisoning
if
inhaled in excess
Erythrocytes:
Contain
hemoglobin
molecules
Have no antigens
present on the
surface
From a healthy Are larger
individual
are leucocytes
hemolysed when
put in isotonic
saline
Which of the following factors is likely to cause a Change
of An increase in Blood transfusion
significant increase in production of red blood cells:
residence to the physical activity
high mountains
The major function of red blood cells is:
To
transport To destroy the To
form
hemoglobin,
invading bacteria immunoglobulins
which
caries by the process of
oxygen from the phagocytosis
lungs to the tissues
and carbon dioxide
from the tissues to
the lungs
Hemoglobin carries carbon dioxide in the form of:
Carboxyhemoglob Oxyhemoglobin
Carbhemoglobin
in
The followings are the principal modulators of oxygen Mg2+
H+ concentration
Temperature
affinity of haemoglobin except:
A raised ESR will be seen in the following conditions Hysteria
Pregnancy
Pulmonary
except:
tuberculosis
Ig G has all following except:
Is important in Has a molecular Is the principal
mucosal immunity weight of 150000
immunoglobulin
in
secondary
immune response
A child falls and bumps her head on the floor Tissue Factor VII
Factor V
factor is exposed beneath the endothelium of
traumatized blood vessels Which of the following
procoagulant proteins binds to tissue factor and initiates
the clotting cascade?
During the passage of an intravenous catheter, Von Willebrand Factor VIII
numerous endothelial cells are dislodged from the lining factor
of the popliteal vein. What substance allows platelet
Bacterial infection
Hemolytic disease
To
promote To
prevent
coagulation of the coagulation of the
blood
blood
Methemoglobin
Myoglobin
CO2
2,3-DPG
Exercise
Factor X
Myocardial
infarction
Is
the
most Is
the
only
common
immunoglobulin
circulating
capable
of
immunoglobulins
crossing
the
in the serum
placenta
Fibrinogen
Prothrombin
Factor IX
Fibronectin
Tissue factor
528.
7
529.
7
530.
7
531.
7
532.
7
533.
7
534.
7
535.
7
536.
7
537.
7
538.
7
539.
7
adhesion to the exposed collagen fibers?
In the clotting process, as the hemostatic plug develops,
fibrin polymerizes into monomeric threads which are
held together by noncovalent bonds Which clotting
protein increases the strength of the clot by crosslinking
the newly formed fibrin threads?
Which of the following enzymes is ultimately
responsible for the formation of fibrin monomers
Warfarin (coumarin) is an anticoagulant that is often
given to patients following a heart attack However, if
too much warfarin is administered, the patient can have
episodes of bleeding The bleeding produced by
warfarin can be overcome by administering which of
the following
There will be a decrease in neutrophil count in the
following conditions except:
Factor XIII
High
molecular Plasminogen
weight kininogen
(HMWK)
Thrombin
von
Willebrand
Factor (vWF)
Thrombin
Heparin
Plasminogen
Kininogen
Prothrombin
Vitamin K
Aspirin
Heparin
t-PA
(tissue Fibrinogen
plasminogen
activator)
When drugs are In
pernicious In response to During
an
used to suppress anaemia
trauma
overwhelming
the bone marrow
bacterial infection
The most important function that neutrophils and Phagocytosis
Coagulation
Urea absorption
Antibody
monocytes carry out is:
formation
Monocytes of the blood:
Best macrophages Originate
from Give
rise
to Can manufacture
precursor cells in plasma cells
immunoglobulin
lymph nodes
The quantity of thrombin formed under physiological Thromboplastin
conditions depends upon the presence of:
The best index of platelet function is:
Bleeding time
Which one of the following local factors is released Serotonin
from platelets to check bleeding:
Different chemical substances cause leukocytes to move Chemotaxis
either close to them or away from them, this
phenomenon is known as:
Which one of the following statements is incorrect:
Antigens are not
always protein in
nature
Serotonin
Epinephrine
Plasma
During some stage
of enteric fever
Tachyphylaxis
May migrate to the
tissues to form
reticuloendothelial cells in
spleen, liver and
lymph nodes
Plasminogen
Clot
retraction Clotting time
time
Bradykinin
Histamine
Thromboplastin
generation
Kalikrein
Prothrombin time
Diapedesis
Chemotropism
Hydrotropism
Inflammatory
response
The
antigen
responsible for Rh
grouping is called
as D factor
Some
antigen
antibody reactions
require presence of
plasma
factor
(complement)
The
antigens
responsible for A
and
B
blood
groups
are
unconjugated
proteins
The absence of anti B and anti Rh agglutinins in the III (B) positive or III (B) negative or III (B) positive, IV Rh positive
plasma means the subject is:
IV (AB) positive
IV (AB) negative
(AB) positive, III
Thrombosthenin
Body will readily
form
antibodies
against
foreign
substances having
molecular weight
less than 10000
Type I (0)
540.
7
541.
7
542.
7
543.
7
544.
7
545.
7
546.
7
547.
7
548.
7
549.
7
550.
7
(B) negative or IV
(AB) negative
Platelets have all of the following except that they:
Do
not
have Are formed in the Are non-nucleated Show
contractile power
bone marrow
adhesiveness
while coming in
contact
with
collagen
Neutrophil cells show all the following characteristics Cannot leave the Are motile
Contain lysosomes Are
attracted
except that they:
circulation
chemically to sites
of inflammation
Antihaemophilic globulin and other plasma cofactors Formation
of Activation
of Convertion
of Stabilisation
of
are essential for:
prothrombin
prothrombin
prothrombin
to fibrin
activator
thrombin
Avitaminosis K causes decreased production or Fibrinogen
Prothrombin
Factor VII
Factor IX
decreased plasma concentration of the followings
except:
Which one of the following combination of substances Factor V, VII, Prothrombin,
Prothrombin,
Thrombin
and
2+
present in plasma cause the production of clot:
platelet factor III, factor V, factor tissue factor, Ca , fibrinogen
Ca2+, flbrinogen
VIII,
platelet factor V
factor
A person with blood group B:
Can not be a father Can not be a father An not be a father Has the most
of a child with an of a child with 0 of a child with A common
blood
AB group if the blood group
blood group
type
mother belongs to
0 group
Which one of the following would be potentially An Rh negative Transfusion of one An Rh negative An Rh positive
dangerous situation:
mother who is unit of 0 negative mother who is mother who Is
bearing her second blood to B positive bearing her first bearing her first
Rh positive child
recepient
Rh positive child
Rh negative child
An individual with blood group II(A1) will:
Never agglutinate Not
develop Usually
Usually
the
erythrocytes plasma agglutinins agglutinate
the agglutinate
the
from group A1 of type B blood in erythrocytes from erythrocytes from
donor
the absence of 0 group donor
AB donor
exposure to a B
agglutinogen
Humoral antibody formation response is shown by:
B- lymphocytes
Large
T lymphocytes
Monocytes
lymphocytes
Which of the following decrease platelet aggregation & PGI2
PGE2
PGF2alpha
TBXA2
cause vasodilatation?
Immunoglobulin specificity is determined by:
Variable
heavy Which
enable Constant
heavy Constant light and
and light chain
antigen specificity and variable light variable
heavy
chain
chains
Liberate platelet
factor which helps
in clotting
Phagocytose
foreign particles
Aggregation
platelets
of
Stuart-Prower
factor
Prothrombin,
factor
V,
Fibrinogen, Ca2+
All of the above
statements
are
correct
An Rh positive
mother who is
bearing her second
Rh negative child
Usually
agglutinate
the
erythrocytes from
type A1 donor
None of the above
PGD2
Constant
chains
both
551.
552.
8
8
The ejection fraction equals
The upstroke of the SA nodal action potential is
produced by opening a channel that is
The channel responsible for the initiation of phase-4
depolarization in SA nodal cells
0.60
Primarily
permeable to Na+
Is
primarily
permeable to Na+
553.
8
554.
0.50
Primarily
permeable to Ca2+
Is
opened
by
membrane
depolarization
Could explain why
the output of left
and
right
ventricles is equal
in the long term
8
Starling's law of the heart:
555.
8
Myocardial contractility:
556.
8
The absolute refractory period in the heart:
557.
8
If the end diastolic volume increases:
558.
8
559.
8
During cardiac cycle immediutely after closure of AV
valves there is:
Immediately after closure of semilunar valves, there is:
560.
8
The phase of atrial systole coincides with ventricular:
561.
8
The closure of AV valves is initiated by:
States that strength
of
myocardial
contraction is a
function of initial
length
of
the
muscle fibers
Is enhanced by Doesnot depend
local application upon
an
of epinephrine
interaction
between actin and
myosin filaments
Is the phase of the Is longer than the
cardiac cycle when refractory period
the heart cannot be in the skeletal
excited by any muscles
stimulus how so
ever strong it may
be
Stroke volume is The
force
of
increased
contraction
of
heart decreases
Isometric
Isotonic
contraction
contraction
Isovolumetric
Isometric
relaxation
contraction phase
Isometric
Isotonic relaxation
relaxation
AV
pressure Atrial contraction
gradient
is
maximal
562.
8
During ejection of blood from the ventricles, rate of
flow is:
563.
8
Second heart sound differs from the first in that:
0.55
Primarily
permeable to K+
Is opened by vagal
nerve stimulation
0.65
Primarily
permeable to Cl−
Is
primarily
permeable to K+
0.70
Equally permeable
to Na + and K+
Is
closed
by
norepinephrine
Could explain fall
in cardiac output
when a person
changes
his
posture from lying
down to standing
Is enhanced by
local application
of acetylcholine
Explain
mechanism
extracardial
control
Lasts
approximately as
long as the cardiac
contraction
Corresponds
in Is the same with
time with the relative refractory
duration of the period of the heart
action potential
the Explain
of mechanism
homeometric
control
the
of
Is present when Increased
bathing medium is deficiency of ATP
calcium free
Cardiac output is Starling's law is HR is decreased
decreased
unapplicable
Isometric
relaxation
Itonic relaxation
phase
Isotonic
contraction
Ventricular
contraction
Isotonic relaxation
Isotonic
contraction
Isometric
contraction
Ventricular
relaxation
Protodiastolic
period
Ejection period
Ventricular
contraction
Backward flow of
blood
during
ventricular
contraction
Highest
during Highest in the last Highest in the Roughly
equal Variable
first third
third
middle third
throughout
depending on heart
rate
It is due to It has a higher Its duration is It is occasionally It heard best over
564.
8
In the heart:
565.
8
Electrical conduction in the heart:
566.
8
Rate of ventricular filling:
567.
8
Sympathetic stimulation to the heart would cause:
568.
8
During the process of inspiration in children:
569.
8
Force of contraction of the heart:
570.
8
571.
8
Activity in the sympathetic nerves to the heart increases
in the following except:
Stimulation of sympathetic nerves to the heart causes:
572.
8
The split second heart sound in healthy individual:
573.
8
The murmurs may be detected using a stethoscope over:
574.
8
Ventricular filling:
turbulence set up
by
the
valve
closure
Ventricle shortens
from base to apex
in order to reduce
its diameter
Is slowest in its
transit from SA
node to AV node
frequency
greater than that of split
the first
Contraction begins Excitation process AV
node
is
normally in the can spread directly primary
left atrium
from
atria
to pacemaker of the
ventricles
heart
Is
due
to Is
faster
in Is slowest in its
protoplasmic
ventricular muscle transit through the
bridges between than atrial muscle
AV node
adjacent cardiac
cells
Variable
Is
maximum Is
maximum Is
maximum
depending upon during first third during
middle during last third of
the timing of atrial of diastole
third of diastole
diastole
contraction
Positive
Negative
Negative inotropic Negative
bathmotropic
chronotropic effect effect
dromotropic effect
effect
Pulse
rate Pulse
rate Pulse
strength Pulse rate does not
increases
decreases
increases
change
Increases
with Decreases
by Is unaffected by Decreases
by
sympathetic
athletic training
nutrition
increased filling
stimulation
When the arterial During exercise
During excitement When the arterial
pressure falls
pressure rises
Shifts
the Decrease in the Fall in coronary Decrease in the
Starling's curve of rate of diastolic blood flow
rate of conduction
myocardial
depolarisation in
in AV node
contractility to the pacemaker
right
Is best heard in Is most easily Tends to disappear Is best heard in
pulmonary area
heard
during during inpiration
apical area
expiration
Heart region in
people suffering
from
severe
anaemia
Begins
during
isometric
the apical part of
the heart
Purkinje fibres has
the
lowest
conduction
velocity
Is fastest in its
transit from SA
node to AV node
Is
constant
throughout
diastole
Fall in coronary
blood flow
Pulse
strength
decreases
Is not dependent
upon rate
During stress
Shifts
the
Starling's curve of
myocardial
contractility to the
left
Is usually due to
pulmonary valve
closure
just
preceeding aortic
valve closure
Vessels
having Aneurysmal
The
heart
in During
laminar flow
dilatations in the healthy
young tachycardia
arteries
adults
in
late
diastole
Depends mainly Gives rise to third Does not induce Will not occur if
on
atrial heart sound in tension
on arterial pressure is
relaxation phase
575.
8
Cardiac index is cardiac output:
576.
8
Increased mean arterial blood pressure on sympathetic
stimulation cannot be possibly attributed to:
577.
8
578.
8
579.
8
580.
8
581.
8
Increasing vagal stimulation of the heart will cause an
increase in which of the following?
Sudden standing evokes the baroreceptor reflex. Which
one of the following will be greater after a person
suddenly stands up than it was before the person stood?
Which one of the following will increase if massaging
the neck stretches the carotid sinus baroreceptor:
A healthy 28-year-old woman stands up from a supine
position Which of the following cardiovascular changes
is most likely to occur?
Sympathetic stimulation of the heart results in which of
the following:
582.
8
583.
8
584.
8
585.
8
586.
8
contraction
some people
Per unit surface Per unit body
area
weight
Decreased vagal Increased venous
tone
return to heart
causing increased
cardiac output
PR interval
Heart rate
The
fraction
Per unit
volume
Increased
peripheral
resistance
body Per unit time
Increased
venomotor tone
Ventricular
Ejection fraction
contractility
ejection The end-diastolic The renal blood The venous return
volume
flow
Vagal
activity
Increased
rate
nerve Total
peripheral
resistance
heart Decreased
myocardial
contractility
An increase in the An increase in the
activity of the duration of systole
sarcoplasmatic
reticulum calcium
pump
Hypertension
Tachycardia
Which of the following conditions causes pulse pressure
to increase?
In running 100 metres, the increased oxygen Increased cardiac Increased
requirements of tissues is immediate responses:
output
DPG
2,3
2,5 litres
higher
than
atmospheric
None of the above
More
forceful
cardiac contraction
Cardiac output
The pulse pressure
Right
atrial Venous tone
pressure
Decreased
total Dilation of large
peripheral
veins
resistance
An increase in the A decrease in the
duration
of affinity of troponin
diastole
for calcium
Ventricular
contractility
Increased
renal
blood flow
Hemorrhage
Heart failure
– Increased
erythropoietin
Aortic stenosis
Rise in CO2 partial
pressure activating
peripheral
chemoreceptors
The intrinsic rate Cardiac
output
of
the
heart's increases
with
pacemaker is 100 increased
heart
beats/min
rate
According to the Frank-Starling mechanism of the Stroke
volume The left ventricle
heart:
increases
with ejects a larger
increased venous volume of blood
return
with each systole
than the right
ventricle
Which of the following does not contribute to increased Increased length of Increased
Increased
stroke volume during exercise?
filling time during contractility
of return
diastole
cardiac muscle
Cardiac output in adult man under basal condition 5,0 litres
would be close to:
ventricular muscle
7,5 litres
A decrease in the
concentration of
Ca2+ during systole
Increased arterial
CO2
partial
pressure leads to
vasodilatation
Both
ventricles
contract
simultaneously
venous Increased
Increased
endsympathetic
diastolic volume
stimulation
of
ventricular muscle
9,5 litres
4,0 litres
587.
8
588.
8
589.
8
590.
8
591.
8
592.
593.
8
8
594.
8
595.
8
596.
8
597.
8
598.
8
599.
600.
8
8
601.
8
602.
8
Carotid sinus stimulation causes:
An increase in A fall in venous
heart rate
pressure
The most important cardiac and vasomotor centres are Medulla
Cerebral cortex
located in:
The first heart sound is mainly due to:
Closure of AV Closure
of
valves
semilunar valves
The heart rate increases in the following states except
during:
The pulse pressure would be greatest in:
Process of repolarisation is best demonstrated by:
If an individual has a heart rate of 150, what is the
amount of time it takes to complete one heart beat?
The sino-atrial node is
Hypothyroldism
Dorsalis
artery
T wave
0,4 seconds
Sympathetic
stimulation
paedis Aorta
Q wave
0,8 seconds
Reflex bradycardia
Reflex hyperpnoea
Cerebellum
Hypothalamus
The rapid inflow
of blood from
auricle to ventricle
Muscular activity
Femoral artery
Inflow of blood
from ventricle to
aorta
Emotional
excitement
Femoral vein
Р wave
8 seconds
R wave
150 seconds
Found in the wall A cardiac lymph Located in the
of the right atrium node
ventricular septum
near the opening
between the right
of the superior
and left ventricles
vena cava
Which of the following rhythms is initiated by the AV- 40-50 /min
10-20 /min
60-80 /min
node
The region of the heart having the lowest inherent Purkinje fibres
SA node
AV node
rhythmicity is:
Which of the following is a characteristic of cardiac Cell
fibers more than one cells arranged in
muscle?
arranged in a nucleus per cell
very
long,
branching network
cylindrical fibers
or syncytium
The "wave of depolarization", which represents the Sino-atrial node to Sino-atrial node to A-V bundle to S-A
electrical impulse traveling through the heart's chambers atrio-ventricular
atrial lymph to node to A-V node
with each beat of the heart, follows a certain pathway node to bundle of ventricular lymph
Which of the following pathways is at least partially HIS
node
correct?
The most rapidly propagating tissue in heart is:
Purkinje fibres
Ventricular tissue
Atrial muscle
The "lubb-dup" sounds have practical clinical value Efficiency of heart strength of arterial power of papillary
because they provide information concerning the
valves
contractions
muscles
Cardiac output is:
The product of the The product of the The product of the
stroke volume and pulse
pressure heart rate times the
the heart rate
times the stroke pulse rate
volume
On an "ECG" the spread of an impulse over the atria the "P" wave
the QRS complex
the Q portion of
just prior to atrial contraction is represented by:
the QRS complex
Reflex increase in
venous pressure
Pons
Turbulent flow of
blood in aorta
Stress reacton
Capillaries of the
foot
S wave
60 seconds
Located in the Present only in the
wall of the carotid fetus
artery
80-100 /min
20-30 /min
Atrial muscle
Ventricular muscle
cells located in controlled
walls of arteries voluntarily
and veins
S-A
node
to Purkinje fibers to
Purkinje fibers to bundle of HIS to
bundle of HIS
A-V node to S-A
node
AV node
closure of foramen
ovale
Directly related to
venous return and
Starling's law of
heart
the T wave
Bundle of HIS
all of the above
The product of the
systolic
times
diastolic
blood
pressure
combination of the
RS component of
603.
8
If the cardiac cycle takes only 0,4 seconds, the heart rate 150
will be
Duration of ventricular systole
0,33 sec
During isovolumic relaxation
All valves
closed
604.
605.
8
8
606.
607.
8
8
The interval P-Q lasts (sec)
The repolarisation of the atria:
608.
8
In a typical capillary
609.
8
The ultrafiltration of fluid from the capillaries into the
interstitial spaces increases as the
610.
8
611.
8
612.
8
613.
8
614.
8
615.
8
616.
617.
8
8
75
0,5 sec
are The aortic valve is
closed and the
mitral valve is
open
0,12-0,21
0 6-0,12
Is seldom seen on Is represented by
the ECG
Q wave
37
100
0,4 sec
The aortic valve is
open
and
the
mitral valve is
closed
0,18-0,24
Is represented by
U wave
0,65 sec
The
pulmonic
valve is open and
the tricuspid valve
is closed
0,24-0,27
Is represented by
descending limb of
P wave
The tension in the The pressure at the The
radius The radius is more
wall increases as venous
end exceeds that of a than three times as
the
hydrostatic exceeds that at the typical arteriole
great as the radius
pressure rises
arterial end
of a red blood cell
the QRS complex
240
0,8 sec
The
pulmonic
valve is closed and
the tricuspid valve
is open
0,27-0,32
Has no electrical
properties
The tension in the
wall
is
proportional to the
fourth power of
the radius
Radius of the Molecular weight
the upstream arteriole of the plasma
decreases
proteins decreases
Tissue hydrostatic Oncotic pressure Hydrostatic
pressure decreases of the interstitial pressure in
fluid decreases
capillaries
decreases
The pressure drop in the systemic circulation is greatest Arterioles
Capillaries
Venules
in the
The less compliant arterial system in old people Requires the heart Allows blood to Prevents
the
compared with young people
to expend more flow through the arterial
systolic
energy to pump a capillaries at a pressure
from
normal
cardiac more steady rate
attaining
output
excessively high
values
Which of the following would not influence venous Exchange
of Contraction
of Higher pressure in
return:
materials
at skeletal muscles capillaries than in
capillary bed
squeezing veins
veins,
forcing
blood toward heart
The is the major force pushing fluid out of the Capillary
blood Force of gravity
Respirations in the
capillaries
pressure
body
The "pressure gradient" is the highest or steepest along Arterioles
Large arteries
Capillaries
the:
In a normal person, most of the circulating blood is Systemic veins
Systemic
Pulmonary
located in the
capillaries
capillaries
Velocity of blood flow is greatest in the
Aorta
Capillaries
Venules
The principal determinants of mean arterial pressure are Cardiac output and Arterial
and Cardiac output and
Aorta
Venae cavae
Increases the total Decreases
the
blood volume
mean
arterial
blood pressure
Gravity
Dilation of veins
Valve in a vein
Colloidal osmotic
pressure
Venae cavae
Veins
Pulmonary veins
Venae cavae
Peripheral
Systemic arteries
and arterioles
Arterioles
Cardiac output and
peripheral
resistance
618.
8
619.
8
620.
8
621.
8
622.
8
623.
8
624.
8
625.
8
626.
8
627.
8
628.
8
629.
8
630.
8
resistance
and
arterial
compliance
If arterial blood pressure falls markedly, which of the Less movement of Body cells swollen Leakage of blood Exchange of water
following would you expect to occur?
fluid from the with excess fluid
proteins
into between the blood
capillaries to the
interstitial fluid
and
interstitial
interstitial spaces
fluid unaltered
Which of the following has the highest total cross- Capillaries
Arteries
Arterioles
Venules
sectional area in the body?
Tissue fluid is found in the:
Interstitial spaces
Vascular spaces
Intracellular
Cells
spaces
In a test subject, oxygen consumption was measured at 10,0 L/min
30,0 L/min
4,2 L/min
7,0 L/min
700 mL/min Pulmonary artery oxygen content was 140
mL per liter of blood and brachial artery oxygen content
was 210 mL per liter of blood Cardiac output was
which of the following?
Which one of the following is the best index of Left
ventricular Mean
arterial Left
ventricular Pulmonary
afterload?
mean
systolic blood pressure
end-diastolic
capillary
wedge
pressure
pressure
pressure
Stroke volume is increased by which of the following?
A decrease in An increase in A decrease in A decrease in
venous
heart rate
coronary
blood contractility
compliance
flow
An increased preload would most likely be caused by an Venous tone
Capillary
Myocardial
Heart rate
increase in which of the following?
permeability
contractility
A patient in heart failure improves markedly after using A decrease in end An increase in A reduction in An increase in
a drug that increases the inotropic state of her heart. diastolic volume
cardiac excitability heart rate
end-diastolic
Which one of the following changes is primarily
pressure
responsible for the improvement in her condition
In a resting, healthy man, which of the following is the 0,6
0,9
0,2
0,3
approximate ejection fraction
Phase-4 depolarization of SA nodal cells is caused by An increase in the A decrease in the A decrease in the A decrease in the
which of the following
flow of sodium flow of chloride activity of the Na- flow of potassium
into the cell
out of the cell
K pump
out of the cell
Blood pressure increases and heart rate decreases in Increased
Hemorrhage
Exercise
Increased
body
response to which of the following
intracranial
temperature
pressure
An ectopic extrasystole caused by a ventricular focus is A
skipped A skipped atrial A larger than Interruption of the
characterized by which of the following
ventricular
contraction
normal force of regular SA node
contraction
contraction
discharge
After a mild hemorrhage, compensatory responses
Venous
venous
compliance
Heart rate
arterial
compliance
Ventricular
Total
venous
compliance
Interstitial edema
Veins
Tissues
12,6 L/min
Total
peripheral
resistance
An increase
afterload
in
Arteriolar tone
An increase
wall thickness
in
0,1
An increase in the
activity of the
Na/Ca exchanger
Exposure to high
altitude
Retrograde
conduction of the
action potential to
the atria
peripheral Coronary
blood
631.
8
632.
8
633.
8
634.
8
635.
8
636.
8
637.
8
638.
8
639.
8
640.
8
641.
8
642.
8
initiated by the baroreceptor reflex keeps blood pressure
at or close to its normal value. Which one of the
following values is less after compensation for the
hemorrhage than it was before the hemorrhage?
Which of the following conditions causes pulse pressure
to increase?
Central venous pressure is increased by which of the
following?
compliance
Hypertension
Decreasing
rate
Tachycardia
contractility
resistance
flow
Hemorrhage
Aortic stenosis
Heart failure
heart Decreasing blood Increasing venous Increasing
total
volume
compliance
peripheral
resistance
Blood flow through an organ would be increased by The hematocrit
The diameter of The number of The
arterial
decreasing which of the following
the arterial vessels open
arterial pressure
vessels
A reduction in carotid sinus pressure would cause a Venous
Heart rate
Myocardial
Total
peripheral
decrease in which of the following
compliance
contractility
resistance
Which one of the following organs has the highest Heart
Brain
Skeletal muscle
Kidney
arterio-venous O2 difference under normal resting
conditions
The percentage of the total cardiac output distributed to The ratio of an The
contractile The magnitude of The magnitude of
any single organ is most dependent on which of the organ's
vascular state of the heart
mean
blood diastolic pressure
following
resistance to total
pressure
peripheral
resistance (TPR)
Which one of the following characteristics is most Preload
Stroke work
Afterload
Peak
systolic
similar in the systemic and pulmonary circulations?
pressure
Which one of the following values is greater in the The
vascular The mean arterial The
arterial The blood flow
pulmonary circulation than in the systemic circulation?
compliance
pressure
resistance
Which one of the following increases during aerobic Cerebral vascular Diastolic
blood Mixed
venous Blood flow to the
exercise
resistance
pressure
oxygen tension
kidney
Flow of fluid through the lymphatic vessels will be Capillary oncotic Capillary pressure Capillary
Interstitial protein
decreased if there is an increase in which of the pressure
permeability
concentration
following?
During exercise, cardiac output is augmented by which Increased
Sympathetic
Dilation of venous Decreased
endof the following
ventricular
stimulation
of vessels
diastolic volume
contractility
resistance vessels
A 65-year-old male visits his family practitioner for a Arterial
Cardiac output
Myocardial
Stroke volume
yearly examination. Measurement of his blood pressure compliance
contractility
reveals a systolic pressure of 190 mm Hg and a diastolic
pressure of 100 mm Hg. His heart rate is 74/min and
pulse pressure is 90 mm Hg. A decrease in which of the
following is the most likely explanation for the high
pulse pressure?
Decreasing plasma
aldosterone
concentration
The diameter of
the venous vessels
Cardiac output
Stomach
The magnitude of
cardiac output
Blood volume
The sympathetic
tone
Circulating blood
volume
Central
venous
pressure
Decreased mean
systemic arterial
pressure
Total
peripheral
resistance
643.
8
644.
8
645.
8
646.
8
647.
8
648.
8
649.
8
650.
8
651.
8
A 54-year-old male is seen in clinic with complaints of
palpitations and light-headedness Physical examination
is remarkable for a heart rate of greater than 200 beats
per minute and a blood pressure of 75/40 mm Hg. What
adjustments have probably occurred in the cardiac
cycle?
In a tissue capillary, the interstitial hydrostatic pressure
is 2 mm Hg, the capillary hydrostatic pressure is 25 mm
Hg and the interstitial oncotic pressure is 7 mm Hg. If
the net driving force across the capillary wall is 3 mm
Hg favoring filtration, what is the capillary oncotic
pressure?
Which of the following vascular structures contains the
largest proportion of the total blood volume in a normal
individual?
During normal diastole, which of the following is most
important in preventing over-distension of the
ventricles?
The table above shows the fall in blood pressure that
occurs for the various types of blood vessels as blood
flows from the aorta (100 mm Hg) to the right atrium (0
mm Hg). Which of the following types of blood vessel
is likely to have the highest ratio of wall cross-sectional
area to lumen cross-sectional area?
A 60-year-old male with heart disease is brought to the
emergency room. Cardiovascular evaluation reveals a
resting O2 consumption of 200 mL/min, a peripheral
arterial O2 content of 0.20 mL O2/ml of blood, and a
mixed venous O2 content of 0.15 mL O2/mL of blood.
What is his cardiac output?
A healthy 28-year-old woman stands up from a supine
position. Which of the following cardiovascular
changes is most likely to occur?
During an experimental procedure, a cardiovascular
researcher prepares his anesthetized animal subject for
blood pressure and electrocardiogram monitoring. He
then
isolates
and
electrically
stimulates
glossopharyngeal afferent fibers that supply the carotid
sinus. Which of the following changes would most
likely occur in this subject?
In which type of blood vessel is the mean linear velocity
of a red blood cell the lowest?
Systolic time has
decreased
but
diastolic time has
decreased more
Diastolic time has Diastolic time has
decreased
and decreased
but
systolic time has systolic time has
increased
decreased more
Systolic time has Systolic time has
decreased
and decreased
but
diastolic time has diastolic time has
increased
not changed
27 mm Hg
21 mm Hg
23 mm Hg
24 mm Hg
25 mm Hg
Venules and veins
Aorta and large Arterioles
arteries
Capillaries
Pulmonary
vasculature
Fibrous
pericardium
Adjacent lungs
Diaphragm
Mitral valve
Arterioles
Aorta and large Small arteries
arteries
Capillaries
Venules and small
veins
4.0 L/min
2.5 L/min
25.0 L/min
100.0 L/min
Aortic valve
10.0 L/min
Increased
rate
heart Decreased
Decreased
total Dilation of large Increased
renal
myocardial
peripheral
veins
blood flow
contractility
resistance
Hypotension with Hypertension with Hypertension with Hypotension with No changes in
bradycardia
bradycardia
tachycardia
tachycardia
blood pressure or
heart rate
Capillaries
Aorta and large Arterioles
arteries
Small arteries
Vena cavae and
large veins
652.
9
Out of the twenty three generation of passages from
conducting zone of airways, gas exchange occurs in:
The exchange of gases between the lungs and lung
capillaries is called:
Dalton's Law describes:
653.
9
654.
9
655.
656.
9
9
The partial pressure of oxygen in dry air at sea level:
The intrapleural pressure during quiet breathing at the
start of inspiration:
657.
9
The oxygen required by the respiratory muscles would
be increased by all the following except:
658.
9
Two healthy women with identical tidal volumes and
respiratory rates are subjected to spirometry and blood
gas measurements Subject A doubles her tidal volume
and decreases her respiratory rate to one-half of
baseline. Subject B decreases her tidal volume to onehalf of baseline and doubles her respiratory rate. Which
of the following statements about the resulting alveolar
ventilation in the two women is true?
The concentration of CO2 is lowest in:
659.
9
660.
9
661.
9
662.
9
663.
9
664.
9
7 generation
1 generation
3 generation
external
internal respiration ventilation
respiration
the laws of partial the principles of the
laws
of
pressures of gases expansion
temperature
difference between
partial gases
159 mmHg
163 mmHg
149 mmHg
Decreased from - Increased from 2,5 Decreased from 6
2,5 mmHg to -6,0 mmHg to 6,0 mmHg to 2,5
mmHg
mmHg
mmHg
A decrease in A decrease in lung An increase in the
airway resistance
compliance
rate of respiration
Alveolar
ventilation
increases
subject A
decreases
subject B
15 generation
breathing
perfusion
the principles of
temperature
differences under
water
100 mmHg
Increased from -6
mmHg to -2,5
mmHg
A decrease in the
production
of
pulmonary
surfactant
Alveolar
Alveolar
Alveolar
ventilation
is ventilation
ventilation
in unchanged in both increases in both decreases in both
and subjects
subjects
subjects
in
the anatomic dead the anatomic dead the alveoli at end
space
at
end space
at
end inspiration
inspiration
expiration
Functions of alveolar macro-phages include all the secretion
of phagocytosis
of release
of
following except:
surfactant
bacteria
lysosomal
enzymes into the
alveolar space
Peripheral and central chemo-receptors may both an increase in a
decrease in a
decrease in
contribute to the increased ventilation that occurs as a arterial
carbon arterial
oxygen arterial
blood
result of:
dioxide tension
content
pressure
The water vapor pressure of alveolar gas at a barometric 47,0 mmHg
23,5 mmHg
76,0 mmHg
pressure of 380 mmHg is:
The oxygen required by the respiratory muscles would A decrease in A decrease in lung An increase in the
be increased by all the following except:
airway resistance
compliance
rate of respiration
The concentration of CO2 is lowest in:
5 generation
the principles of
gas exchange
50 mm Hg
Increased from -6
mmHg to 2,5
mmHg
An increase in
tidal volume
Alveolar
ventilation
decreases
subject A
increases
subject B
in
and
in
the alveoli at end the blood in the
expiration
pulmonary veins
transport
of release
inhaled particles leukocyte
out of the alveoli
chemotactic
factors
a
decrease in an increase
arterial
oxygen arterial pH
tension
94,0 mmHg (E)
105,0 mmHg
of
in
A decrease in the An increase in
production
of tidal volume
pulmonary
surfactant
the anatomic dead the anatomic dead the alveoli at end the alveoli at end the blood in the
665.
9
Which one of the following would increase in
obstructive, but not in restrictive, pulmonary disorders?
Very small particles are removed from the respiratory
system by which of the following:
Which one of the following is higher at total lung
capacity than it is at residual volume?
666.
9
667.
9
668.
9
669.
9
670.
9
671.
9
672.
9
673.
9
674.
9
The activity of the central chemoreceptors is stimulated
by which of the following?
675.
9
Which one of the following is the most likely cause of a
high arterial PCO2:
676.
9
677.
9
Which of the following will increase as a result of
stimulating parasympathetic nerves to the bronchial
smooth muscle?
An increase in altitude causes a decrease in gas
exchange efficiency because:
A spirometer can be used to directly measure which of
the following:
A woman has a respiratory rate of 18, a tidal volume of
350 mL, and a dead space of 100 mL What is her
alveolar ventilation:
Chronic cigarette smoking can produce all the following
except:
space
at
end
inspiration
Functional
residual capacity
Phagocytosis
Anatomical
space
Inspiratory
capacity
4,5 L
space
at
end inspiration
expiration
Vital capacity
Maximum
breathing capacity
Bulk flow
Diffusion
dead Maximum static Lung compliance
inspiratory
pressure
Functional
Residual volume
residual capacity
4,0 L
5,0 L
Decreased
Emphysema
anatomic
dead
space
Enzymes within the lung are responsible for the Angiotensin II
Bradykinin
activation of which of the following?
A respiratory acidosis that results in an increase in the Stimulate
the Decrease
the
concentration of hydrogen ion in arterial blood from 40 peripheral
amount
of
meq/L (pH 7.4) to 50 meq/L (pH 7.3) would do which chemoreceptors
ammonium
of the following:
excreted in the
urine
Which of the following will return toward normal Arterial hydrogen Arterial
carbon
during acclimatization to high altitude?
ion concentration
dioxide tension
Atelectasis
Prostaglandins
expiration
pulmonary veins
FEV1
Expectoration
Breathing
frequency
Ciliary transport
Airway resistance
Alveolar pressure
Total
capacity
5,5 L
lung Physiological dead
space
6,0 L
Carbon
monoxyhemoglobi
nemia
Serotonin
“Morning
coughing”
Leukotrienes
Inhibit the central Increase the pH of Decrease
the
chemoreceptors
the urine
concentration of
HCO3– in arterial
blood
Arterial
bicarbonate
ion
concentration
An increase in the A decrease in the A decrease in the
PCO2 of blood PO2 of blood oxygen content of
flowing through flowing through blood
flowing
the brain
the brain
through the brain
Depressed
Increased
Increased alveolar
medullary
metabolic activity dead space
respiratory centers
Bronchoconstrictio Lung compliance
Elastic work of
n
breathing
Arterial
hemoglobin
concentration
A decrease in the
metabolic rate of
the
surrounding
brain tissue
Alveolar capillary
block
Alveolar
ventilation
Bronchodilatation
Anatomic
space
There is a decrease Less
blood The surface area
in
oxygen circulates through of the alveoli
pressure, therefore the lungs
decrease
less of a gradient
There are fewer
red blood cells in
the blood at high
altitudes
At high altitudes,
evil spirits inhibit
vital processes of
the body
An increase in the
pH of the CSF
Increased alveolar
ventilation
dead
into the lungs
during inspiration
Hyperventilation in response to a stressful situation An increase in the
leads to which of the following?
activity of the
central
chemoreceptors
Pulmonary arteriolar constriction can be caused by all Increase in pH
the following except:
(local)
678.
9
679.
9
680.
9
681.
9
682.
9
683.
9
684.
9
685.
9
686.
9
687.
10
688.
689.
10
10
690.
10
Metabolic rate is increased least with:
691.
10
A man with a respiratory quotient (RQ) of 70:
Pulmonary arteries are dilated by all the following
except:
The respiratory rate after exercise does not reach basal
level until:
The clinical sign of cyanosis is caused by which of the
following:
A decrease in the
excitability
of
nerve and muscle
cells
None of all
Adrenaline
A decrease in the A decrease in pH An increase in the
blood flow to the of the arterial resistance of the
brain
blood
pulmonary blood
vessels
Local
O2 Local
Systemic hypoxia
deficiency
accumulation of
CO2
Acetylcholine
PGI2
Isoproterenol
O2 debt is repaid
15 min
An increase in the
concentration of
deoxygenated
hemoglobin in the
blood
Carbon monoxide
An increase in the A decrease in the
affinity
of percent of red
hemoglobin
for blood
cells
oxygen
(hematocrit)
PCO2 return
normal
A decrease in
concentration
iron in the
blood cells
30 min
Which one of the following gases diffuses across the
Oxygen
alveoli-capillary membrane by a diffusion-limited
transport process?
Which one of the following will decrease in a person Arterial
carbon Anion gap
with ventilation-perfusion (V/Q) abnormalities
dioxide tension
Nitrogen
Which one of the following is higher at the apex of the
lung than at the base when a person is standing
Ventilation
VentilationBlood flow
perfusion
(V/Q)
ratio
Patients with chronic lung disease are often divided into V/Q ratios
Vital capacities
those who are cyanotic and (those who are not cyanotic
The presence of cyanosis in blue bloaters but not in pink
puffers results from difference in which of the
following?
How many hours after a meal is basal metabolic rate 12 hours
1 hour
(BMR) measured?
The basal metabolic rate is determined primarily:
Thyroxine
Hydrocortisone
The basal metabolic rate (BMR) increases with all the Advancing age
Anxiety
following except:
Arterial pH
90 min
An increase in the
concentration of
carbon monoxide
in the venous
blood
Carbon dioxide
Nitrous
(N2O)
to
the
of
red
oxide
Alveolar-arterial
Alveolar
(A-a) gradient for ventilation
oxygen
PaCO2
Lung compliance
Airway resistances
Total
capacities
2 hours
6 hours
Insulin
Body surface area
NO
lung Expiratory
rates
Growth hormone
Increased
environmental
temperature
Increased
CNS Exercise
Specific dynamic Hot climate
activity
action of food
Has been fasting Has been eating a Has been eating a Has
eaten
18 hours
Epinephrine
Reduced
environmental
temperature
Cold climate
Is dehydrated
flow
for 24 hours
692.
10
693.
10
694.
10
695.
10
696.
10
697.
698.
10
10
699.
10
Which of the following would not produce an increase
in the plasma level of free fatty acids?
700.
10
701.
702.
703.
10
10
10
704.
10
705.
706.
high-fat diet
high-protein diet
Decreased heat production under general anaesthesia is
due to:
A low respiratory quotient in a septic patient is due to:
decreased skeletal decreased anterior
muscle tone
pituitary function
fat metabolism
increased
lactic
acid
The set-point in temperature regulation controls the posterior
anterior
body’s response to changes in temperature The location hypothalamus
hypothalamus
of sensory receptors which regulates the set-point is:
In thermoregulation all statements are true except:
peripheral
brown fat is an
vasoconstriction
important source
decreases
heat of heat production
production
in neonates
vasodilatation
increased
ventilation
spinal cord
skin
41 kcal
1 kcal
shivering is due to
impulses
conducted
via
autonomic
efferents
410 kcal
4 kcal
Metabolism of l
mol of long chain
fatty acid
A
drug
that
inhibits
hormonesensitive
lipase
3kcal
Aerobic
metabolism of l
mol of glucose
A
drug
that
increases the level
of
intracellular
cAMP
5kcal
Anaerobic
metabolism of l
mol of glucose
A
drug
that
activates β3 –
adrenergic
receptors
Which of the following does not increase the output of
glucose from the liver?
Insulin
Induction
phosphorylase
Vitamin B
Growth hormone
Growth hormone
Vitamin A
Hydrocortisone
Thyroxine
10
Which one of the following is a water soluble vitamin?
Somatomedins are required for the anabolic effects of:
Which of the following hormones stimulates anabolism
of proteins and catabolism of fat?
Which of the following daily intakes (in grams) of
carbohydrates (C), protein (P), and fat (F) would be best
for a healthy 80-kg male 40 year construction worker?
All following are true for ingested lipid except:
10
The functions of lipids include all except:
In the body, metabolism of 10 g of protein would
produce approximately:
The energy value of 1g of carbohydrate is:
Which of the following producers the most high-energy
phosphate compounds?
of Glucagon
Vitamin D
Insulin
Insulin
carbohydrates for
24 hours
starvation
decreased Na-KATPase activity
fever
hypoxaemia
great veins
sweating
mediated
sympathetic
cholinergic
neurons
4100 kcal
is respiratory
heat
by loss
is
insignificant under
normal conditions
7kcal
9kcal
Metabolism of l Metabolism of l
mol of glucose
mol of amino acid
41 cal
A
drug
that
decreases
the
metabolic
clearance
of
glucagon
Increased
intracellular
protein kinase A
Vitamin E
Thyroxine
Glucagon
C: 400, P: 80, F: C: 420, P: 80, F: C: 420, P: 130, F: C: 325, P: 80, F:65
65
125
100
Is not important in Increases in the
prostaglandin
faeces with a
synthesis
decrease in bile
secretion
Transporting
Acting as a storage
glucose
across of
metabolic
A
drug
that
inhibits
phosphodiesterase
Epinephrine
Vitamin K
Epinephrine
Epinephrine
C: 340, P:50,F:65
Is absorbed via the Is mainly in the Can be used as a
intestinal
form
of source of ATP
lymphatics
triglycerides
production
Providing
protection
Preventing
for injury
cell Acting
hormones
as
and
membranes
707.
10
In protein deficiency, water accumulates in the Kwashiorkor
intracellular spaces, the condition is known as:
Which one of the following statements about small Contractile
intestinal motility is correct?
activity is initiated
in response to
bowel
wall
distention
Which one of the following statements about gastric Vagotomy
emptying is correct?
decreases
accommodation of
the
proximal
stomach
Vitamin B12 is absorbed primarily in the
Ileum
The principal paracrine secretion involved in the Somatostatin
inhibitory feedback regulation of gastric acid secretion
is
Which one of the following is the putative inhibitory Vasoactive
neurotransmitter responsible for relaxation of intestinal peptide
gastrointestinal smooth muscle?
Cholera toxin causes diarrhea by inhibiting
Cl- absorption in
the small intestine
708.
11
709.
11
710.
711.
11
11
712.
11
713.
11
714.
11
Which one of the following statements best describes
water and electrolyte absorption in the GI tract?
715.
11
716.
11
717.
11
The hormone involved in the initiation of the migrating
motor complex is
The rate of gastric emptying increases with an increase
in
Basal acid output is increased by
718.
11
719.
11
720.
11
reserves
vitamins
Marasmus
organs such as the
kidney
Edema
Anemia
Peristalsis is the
major contractile
pattern
during
feeding
Migrating motor
complexes occur
during
the
digestive period
Vagotomy
abolishes
contractile activity
during
the
digestive period
Vagotomy
Indigestible food Acidification
of
accelerates
the empties during the the
antrum
emptying of solids digestive period
decreases gastric
emptying
Contractile
frequency
is
constant
from
duodenum
to
terminal ileum
Solids empty more
rapidly
than
liquids
Duodenum
Gastrin
Jejunum
Histamine
Stomach
Enterogastrone
Colon
Acetylcholine
Dopamine
Somatostatin
Substance P
Acetylcholine
Electrogenic Na Na-glucose
absorption
from coupled absorption
the small intestine from the small
intestine
The majority of The small intestine Osmotic
absorption occurs and colon have equilibration
of
in the jejunum
similar absorptive chyme occurs in
capacities
the stomach
Motilin
Gastrin
Secretin
Intragastric
volume
Alkalinization
the antrum
Intraduodenal
volume
of Administration of
an H2 receptor
antagonist
Which one of the following processes applies to the Adaptive receptive Peristalsis
proximal stomach?
relaxation
After secretion of trypsinogen into the duodenum, the Enteropeptidase
Procarboxypeptida
enzyme is converted into its active form, trypsin, by
se
The major mechanism for absorption of sodium from Difffusion
Cotransport with
Fat content
duodenum
Vagotomy
Spongy disease
Na/H exchange in Electrogenic Na
the small intestine absorption
from
the colon
Most water and
electrolytes come
from
ingested
fluids
Cholecystokinin
of Osmolality
duodenum
Acidification
the antrum
Water absorption
is independent of
Na
absorption
Enterogastrone
of Acidity
duodenum
of Acidification
the duodenum
Retropulsion
Segmentation
Vill contraction
Pancreatic lipase
Previously
secreted trypsin
Na -H
An alkaline pH
Electrogenic
Solvent drag
of
of
721.
11
722.
11
723.
11
724.
11
725.
11
726.
11
727.
11
728.
11
729.
11
730.
11
731.
11
the small intestine is
Pharmacological blockade of histamine H2 receptors in Inhibits
both
the gastric mucosa
gastrinand
acetylcholinemediated secretion
of acid
Removal of proximal segments of the small intestine Pancreatic enzyme
results in a decrease in
secretion
Dietary fat, after being processed, is extruded from the Chylomicrons s
mucosal cells of the gastrointestinal tract into the
lymphatic ducts in the form of
Gas within the colon is primarily derived from which Fermentation
of
one of the following sources?
undigested
oligosaccharides
by bacteria
Removal of the pyloric sphincter is associated with
An increase in the
rate of gastric
emptying of solids
Removal of the terminal ileum will result in
An increase in the
water content of
the feces
potassium
Inhibits
gastrininduced but not
meal-stimulated
secretion of acid
transport
Has no effect on
either
gastrininduced or mealstimulated
secretion of acid
Maximal
acid Gastric emptying
output
of liquids
Diglycerides
Triglycerides
Vitamins synthesized by intestinal bacteria and Vitamin K
absorbed in significant quantities include
Which one of the following statements about the colon Absorption of Na+
is correct?
in the colon is
under
hormonal
(aldosterone)
control
Contraction of the gallbladder is correctly described by It
occurs
in
which of the following statements?
response
to
cholecystokinin
Folic acid
exchange
Prevents activation Causes an increase
of adenyl cyclase in
potassium
by gastrin
transport
by
gastric
parietal
(oxyntic) cells
Gastric emptying Basal acid output
of solids
Monoglyceride
Free fatty acids
Diffusion from the CO2 liberated by Swallowed
blood
the interaction of atmospheric air
HCO3 − and H
An increase
maximal output
acid
A decrease
absorption
amino acids
in An increase
of basal output
acid
in An increase in
of concentration
bile acid in
enterohepatic
circulation
Thiamine
Air pockets within
foodstuffs
in A decrease in An increase in the
of gastric compliance serum gastrin level
the A decrease in the An increase in the
of fat content of the absorption of iron
the feces
Riboflavin
Vitamin B6
Bile acids enhance Net absorption of Net absorption of The
luminal
absorption
of HCO3- occurs in K+
occurs
in potential in the
water from the the colon
the colon
colon is positive
colon
It is inhibited by It is stimulated by It is inhibited by a It
occurs
the presence of atropine
fat-rich meal
simultaneously
amino acids in the
with
the
duodenum
contraction of the
sphincter of Oddi
Acidification of the duodenum will
Decrease gastric Increase secretion Decrease
Increase
Increase
emptying
of gastric acid
pancreatic
contraction of the contraction of the
secretion
of gallbladder
sphincter of Oddi
bicarbonate
Nearly all binding of cobalamin (vitamin B12) to Duodenum
Stomach
Jejunum
Ileum
Colon
intrinsic factor occurs in the
732.
11
Which one of the following statements about the Absorption may be
process of vitamin B12 absorption is correct?
reduced
in
a
patient
with
gastroectomy
Vitamin B12 binds
preferentially to
intrinsic factor in
the stomach
733.
11
734.
11
735.
11
736.
11
737.
11
738.
11
The origin of electrical slow wave activity in The
interstitial The
smooth
gastrointestinal tract smooth muscle is
cells of Cajal
muscle of the
circular
muscle
layer
The major factor that protects the duodenal mucosa Pancreatic
The endogenous
from damage by gastric acid is
bicarbonate
mucosal barrier of
secretion
the duodenum
Bicarbonate absorption from the upper small intestine is Na/H exchange
Neutral
NaCl
closely coupled with
absorption
The transport protein responsible for entry of glucose SGLT1
Glut-5
into the intestinal enterocyte is called
The following statements are true in relation to saliva Contains
an Has
important
except that it:
enzyme which is lubricating action
essential
for
complete digestion
of lipids
Saliva:
Secretion
is Is primarily under
increased
by hormonal control
parasympathetic
stimulation
739.
11
Which of the following statements is incorrect? Saliva:
Is essential for the Prevents
complete digestion caries
of starch
740.
11
Oesophagus at rest is:
closed at the top open at top
and the bottom
741.
11
Oesophageal peristalsis:
Can occur in the
absence
of
a
swallow
742.
11
The lower oesophageal sphincter:
Has a resting tone
that
maintains
pressure
below
atmospheric
In adults, vitamin
B12
absorption
occurs along the
length of the small
intestine
The
smooth
muscle of the
longitudinal
muscle layer
Duodenal
bicarbonate
secretion
Na-glucose
absorption
Glut-2
Has
action
In
humans,
intrinsic factor is
secreted
from
chief cells of the
gastric gland
The
smooth
muscle of the
muscularis mucosa
Absorption occurs
via
passive
diffusion into the
enterocyte
Hepatic
bicarbonate
secretion
Electrogenic
absorption
SGLT2
Bicarbonate
contained in bile
The
plexus
myenteric
Na Na–vitamin
absorption
SGLT5
B12
antiseptic Maintains mouth Helps in speech
pH around 7
Has
larger
proportion of Na+
in response to
aldosterone
administration
dental Helps
in
deglutition
open at bottom
Has
less Shows none of the
proportion
of above features
ketone bodies in
diabetic ketosis
Is a well buffered
solution that tends
to maintain a pH
of 7 0 in the mouth
open at top and
bottom
Prevents
decalcification of
the teeth
contracted
throughout
its
length
Is abolished by Is under hormonal Moves a bolus by Is not effective for
vagotomy
control
decreasing
liquids
pressure in front of
it
Is a distinct ring of Relaxes before a Is primarily under Consists of striated
circular muscle
bolus arise
hormonal control
muscle
743.
11
Gastric acid secretion is decreased by:
Vagal inhibition
744.
11
Which one of the following statements is incorrect:
745.
11
746.
11
The followings are some of the gastrointestinal
hormones except:
Vitamin B12 deficiency may result from:
747.
11
Which one of the following statement is incorrect:
748.
11
The followings are true for stomach except that it:
749.
750.
11
11
751.
11
752.
11
753.
11
The hormone mainly acting on stomach is:
Hypokalemic metabolic acidosis can occur with excess
fluid loss from which of the following:
Which of the following hormones is involved in the Motilin
initiation of the migrating motor complex:
Patient undergoes a gastrectomy and doctor informs Parietal cells
her that she will need to take vitamin B12 for the rest of
her life Absence of which of the following cell types is
responsible for this vitamin replacement requirement?
Which of the following paracrine secretions is Somatostatin
responsible for inhibiting gastric acid secretion:
Luminal peptides Noradrenaline
& amino acids
M1
cholinergic
antagonist
same
efficacy
at
reducing gastric
acid secretion
Biliary secretion is Daily
salivary Gastric secretion Per day production
2500 ml/day
secretion is about per day is about of pancreatic juice
1500 ml
2000 ml
is about 1500 ml
Chymotrypsin
CCK-PZ
GIP
Motilin
Distension
bowel wall
Total gastrectomy High
dietary Inadequate intake
or ileal resection
intake
of of fresh fruits and
phosphates
green
leafy
vegetables
More proteins in Dietary
fibre An increase in the
the faeces come content improves cellulose content
from bacteria than bulk of faeces
in diet tends to
from diet
cause an increase
in the frequency of
bowel movements
Following
resection
Total gastrectomy
Constipation
is
associated
with
symptoms
of
headache, loss of
appetite
and
restlessness, all of
which are due to
absorption
of
toxins from the
faeces
Shows process of Contains
a
receptive
pacemaker near its
relaxation during cardiac portion
deglutition
Secretes enzymes Responds to an
like
trypsin, increase in its
amylase
contents from 6001600 mL with less
than 5 mm Hg
increase
in
pressure
Gastrin
Pancreozymin
Secretin
Stomach
Ileum
Colon
of
Intestinal
secretions
are
about 1500 ml/day
Villikinin
ileal
A person who
does not defaecate
once a day is not
considered
as
constipated
and
unhealthy
Empties
more
rapidly in response
to liquid than to
solid diet
Bombesin
Pancreas
VIP
Liver
Gastrin
Secretin
Cholecystokinin
Enterogastrone
Chief cells
G cells
Goblet cells
Mucous neck cells
Histamine
Enterogastrone
Pepsin
Enterooxyntin
754.
11
755.
11
756.
11
757.
11
758.
11
759.
11
760.
761.
11
11
762.
11
763.
11
764.
11
765.
11
766.
11
767.
11
The rate of gastric emptying increases with an increase
in which of the following:
Basal acid output is increased by which of the
following?
Intragastric
volume
Alkalinization
the antrum
Which one of the following processes applies to the
proximal stomach:
During a fast, a brief phase of intense sequential
contractions begins in the stomach and gradually
migrates to the ileum. Release of which of the following
intestinal hormones is most likely responsible for this
observed effect?
A patient undergoes a total gastrectomy because of a
proximally located gastric cancer. After the surgery,
which of the following digestive enzymes will be
produced in inadequate amounts?
Withdrawal from chronic administration of an
antisecretory compound is followed by rebound gastric
acid hypersecretion. Which drug could account for the
observed result:
Gastric emptying is slowest after consuming:
Which of the following is the principal paracrine
secretion involved in the inhibitory feedback regulation
of gastric acid secretion:
Patients may experience nausea and a sense of early
satiety following which of the following procedures:
Accommodation
Peristalsis
Motilin
Cholecystokinin
Pepsin
Amylase
A proton
inhibitor
Intraduodenal
volume
of Acidification
the antrum
Fat content of
duodenum
of Administration of
an H2 receptor
antagonist
Retropulsion
of Acidity
duodenum
Acidification
the duodenum
Segmentation
Trituration
Gastrin
Gastrin-releasing
peptide
Secretin
Chymotrypsin
Lipase
Trypsin
pump A H1 receptor A
cholinergic An antacid
antagonist
receptor antagonist
High fat meal
Somatostatin
High protein meal
Gastrin
Surgical resection A vagotomy of the
of the proximal distal stomach
small bowel
Zollinger-Ellison syndrome patients experience severe, Gastrin
Secretin
intractable peptic ulcer disease due to gastric acid
hypersecretion. The cause of the increased acid output
is increased plasma levels of which of the following:
Lactose content of milk for lactose-intolerant people lactase
beta galactosidase
can be decreased using insolubilized:
The carbohydrates in oral cavity are hydrolysed by the Amylase
Lactase
following enzyme:
The ileocecal sphincter:
Is mediated by Relaxes
when
vagovagal reflex
ileum is distended
Small gut contractions:
Osmolality
duodenum
Vagotomy
Are enhanced by Are
absent
parasympahetic
fasted man
activity
of
of
A CCKB receptor
antagonist
Alcohol
Histamine
Prokinetics
Enterogastrone
Surgical resection
of the proximal
stomach
Somatostatin
Surgical removal A vagotomy of the
of the gastric orad stomach
antrum
VIP
Motilin
hexokinase
phosphoglucomuta
se
Sucrase
Maltase
Relaxes
when
proxbial co bn is
distended
h Are
primarity
porpubive h fed
man
Carbohydrates
Acetylcholine
glucokinase
Alpha-dextrinase
Pressure is under Constics
when
hormonal con to I
thepressureb ileum
bcreases
May occur as Are triggered by
migrating motor sbw waves
complex h fed
768.
11
Iron absorption in intestine is:
Dependent upon Enhanced
by
iron excretion
conversbn
of
ferrous to ferr ic
form
The carbohydrates in small gut are hydrolysed by the fo Ptialyn
Lactase
bowing enzymes except:
Succus entericus contains:
A substance which Enzmes
which
activates
break down starch
trypsynogen
769.
11
770.
11
771.
11
Which of the following characterizes bile:
bile
salts
contribute to the
solubility
of
cholesterol in the
bile
772.
11
The liver has all activities except:
773.
11
774.
11
775.
11
776.
11
777.
11
778.
11
After secretion of trypsinogen into the duodenum, the
enzyme is converted into its active form, trypsin, by
which of the following?
Which of the following hormones is most important in
the initiation of gallbladder contraction following a fatty
meal?
Release of which of the following peptides leads to an
increase in the secretion of pancreatic enzymes into the
small intestine?
Patient with episodic abdominal pain notes that the pain
increases after the ingestion of a fatty meal. The action
of which of the following hormones is responsible for
the postprandial intensification of symptoms?
Biochemical analysis of a duodenal aspirate after a meal
reveals a deficiency of enterokinase. The levels of
which of the following digestive enzymes would be
affected?
In fat digestion:
has its highest
oxygen tension at
the centre of a
lobule
Enteropeptidase
Improved
phytates
Maltase
person
by Regulated
h Depressed
by
accord an cevvith gastric secretions
bebody'sneed for
iron
Sucrase
Alpha-dextrhase
Enzymes
which
convert
monosachharides
into
simpler
molecules
contains bilirubin contributes more
mainly in the than
pancreatic
unconjugated form secretion to the
neutralisation of
acid from the
stomach
receives most of produces heparin
its oxygen supply
from the portal
vein
Procarboxypeptida Pancreatic lipase
se
Electrolites having None of be above
similar
concentration as a
blood
CCK
Gastrin
Cholecystokinin
becomes
more
alkaline following
concentration in
the gall bladder
is produced at a
rate
of
approximately
2000 ml/day
has
a
normal
portal
venous
pressure of smaler
than 20 mmHg
Previously
secreted trypsin
receives
approximately
25% of the cardiac
outp
An alkaline pH
GIP
Secretin
VIP
Gastrin
Motilin
Secretin
Somatostatin
Cholecystokinin
Gastrin
Pepsin
Secretin
Somatostatin
Trypsin
Amylase
Colipase
Lactase
Pepsin
Pancreatic lipase Bile salts are the Gastric lipase is Digestion
takes Micelles attach to
in the duodenum is most
efficient the most important place in micelles
enterocyte
the most important emulsifiers
receptor
779.
11
Secretin
11
Release of which one of the following increases the pH
of duodenal contents?
Functions of the liver include all except:
780.
Gastrin
Intrinsic factor
Cholecystokinin
781.
11
Glucagon has all functions except:
stimulates
glycogen synthesis
782.
11
Ptyalin
783.
11
The carbohydrates in small gut are hydrolysed by the
following enzymes except:
In the small intestine, glucose is absorbed:
Passively
By
facilitated
diffusion
Is by facilitated
diffusion
By
co-transport
with chloride
Is less than 10%
dietary intake
Actively by insulin
dependent uptake
Is facilitated by
phosphate
784.
11
Calcium uptake in the intestine:
785.
11
Bacteria in the intestines:
Reduced by the Small intestine is
continuous
sterile
movement
of
contents through
GIT
Occurs by active Occurs with H+
Decreases
with
transport
glucose
Bacteria in small
intestine and large
intestine – same in
number
but
different species
Is
by
active
transport at the
brush
border
membrane passive
across basilateral
membrane
(diffusion)
Fat >CHO>protein
Required for the
breakdown of fiber
786.
11
Na+ absorption in small bowel:
787.
11
788.
11
Speed of delivery of nutrients from stomach to small
intestine:
After ingestion of a meal:
789.
11
790.
11
791.
11
Renin synthesis
In
combination
with sodium
Requires a carrier
protein on the
mucosal side
Reduced in small
intestine due to
gastric acid & fast
motility
CHO>protein>fat
Digestion of fat
and carbohydrate
begins
in
the
mouth
while
protein digestion
begins
in
the
stomach
Which of the following is the major mechanism for Neutral
NaCl
absorption of sodium from the small intestine:
absorption
Absorption of fat-soluble vitamins requires which of the Pancreatic lipase
following:
Bicarbonate absorption from the upper small intestine is Na/H exchange
closely coupled with which of the following?
Gastrin releasing
peptide
Protein
Defence
Glycogenolysis
Homeostasis
metabolism
support
is
a
positive is produced by the stimulates
ts
release
is
inotrope
alpha cells of the production of free increased
in
pancreas
fatty acids in the starvation
blood
Lactase
Maltase
Sucrase
Alpha-dextrinase
Is passive
CHO>fat>protein
Protein>CHO>fat
Carbohydrate in Protein in mouth
the mouth and and
fats
and
protein in the carbohydrate
in
stomach
stomach
Na+-H+ exchange
Intrinsic factor
Na-glucose
absorption
Occurs with Clthrough
tight
junctions
Fat>protein>CHO
Most fluid and Composition
of
electrolytes
are the food has no
absorbed in the effect on transit
large bowel
time through the
bowel
Cotransport with Electrogenic
potassium
transport
Chymotrypsin
Pancreatic
amylase
Neutral
NaCl Electrogenic
absorption
absorption
Solvent drag
Secretin
Na Na-vitamin
absorption
B12
792.
11
The macrolide antibiotic erythromycin is associated
with gastrointestinal disturbances related to increased
motility. The antibiotic works by binding to receptors
on nerves and smooth muscle that recognize which of
the following gastrointestinal hormones:
The principal paracrine secretion involved in the
inhibitory feedback regulation of gastric acid secretion
is:
Which one of the following is the putative inhibitory
neurotransmitter responsible for relaxation of
gastrointestinal smooth muscle?
The paracrine secretion responsible for inhibiting
gastric acid secretion is
In the presence of ADH, the filtrate will be isotonic to
plasma in the
Motilin
Gastrin
Secretin
Cholecystokinin
Enteroglucagon
793.
11
Somatostatin
Gastrin
Histamine
Enterogastrone
Acetylcholine
794.
11
Vasoactive
intestinal peptide
Dopamine
Somatostatin
Substance P
Acetylcholine
795.
11
Somatostatin
Histamine
Enterogastrone
Pepsin
Enterooxyntin
796.
12
797.
12
798.
12
799.
12
Renin secretion by the kidney is increased by
800.
12
801.
12
Nais reabsorbed from the basolateral surface of the
renal epithelial cells by
Which of the following is most likely to cause an
increase in the glomerular filtration rate?
802.
12
Glomerular filtration rate would be decreased by
Compression
of
the renal capsule
803.
12
804.
12
805.
12
The secretion of Hin the proximal tubule is primarily
associated with
Destruction of the supraoptic nuclei of the brain will
produce which of the following changes in urinary
volume and concentration? (Assume that fluid intake
equals fluid loss)
Which of the following statements about renin is true?
Reabsorption
of
bicarbonate ion
An
increased
urinary
volume
and a very dilute
urine
It
converts
angiotensinogen to
Cortical collecting Descending limb
tubule
of the loop of
Henle
Sodium reabsorption from the distal tubule will be Plasma potassium Plasma volume
increased if there is an increase in
concentration
ADH will be released from the posterior pituitary when Plasma volume
Plasma
there is a decrease in
Naconcentration
Increasing
Increasing mean
sympathetic nerve blood pressure
activity
Na-K pump
Na/H exchange
Dilation of the
afferent arterioles
Ascending limb of Medullary
the loop of Henle
collecting tubule
Renal pelvis
Mean
arterial Urine flow rate
pressure
Plasma
Plasma pH
Kconcentration
Plasma osmolality
Plasma
Ca2concentratio
n
Increasing atrial
II natriuretic
hormone secretion
Solvent drag
Increasing
Increasing
glomerular
angiotensin
filtration rate
synthesis
Na-glucose
Facilitated
cotransport
diffusion
Contraction
of Blockage of the Release of renin
mesangial cells
ureter
from
the
juxtaglomerular
apparatus
Constriction of the An increase in A decrease in the
efferent arteriole
afferent arteriolar concentration of
pressure
plasma protein
Excretion
of Excretion
of Reabsorption
of
potassium ion
hydrogen ion
calcium ion
An
increased A normal urinary Decreased urinary
urinary
volume volume
and volume and a very
and concentrated concentration
dilute urine
urine
It is secreted by Its secretion leads Its secretion is
cells
of
the to loss of sodium stimulated
by
Volume depletion
An increase in
renal blood flow
Reabsorption
of
phosphate ion
Decreased urinary
volume and a
concentrated urine
It
converts
angiotensin I to
angiotensin I
806.
12
The effective renal plasma flow, which equals the The
plasma
clearance of PAH, is less than the true renal plasma entering the renal
flow because
vein contains a
small amount of
PAH
807.
12
Most of the glucose that is filtered through the
glomerulus undergoes reabsorption in the
808.
12
809.
12
Which of the following structural features distinguishes The
proximal
the epithelial cells of the proximal tubule from those of tubule has a more
the distal tubule?
extensive
brush
border
Glomerular filtration rate (GFR) and renal blood flow The efferent and
(RBF) will both be increased if
afferent arterioles
are both dilated
810.
12
811.
12
812.
12
Which one of the
aldosterone is correct?
813.
12
The effect of antidiuretic hormone (ADH) on the kidney
is to
814.
12
An increase in the concentration of NaCl in the
intraluminal fluid with the ascending limb of the loop of
Henle causes the macula densa to release
Renin release from the juxtaglomerular apparatus is
inhibited by
Proximal tubule
Increased pressure
within the afferent
arterioles
Patients with renal insufficiency develop very high A
decreased
plasma concentrations of urea (uremia) because of
glomerular
filtration rate
following
statements
about
proximal tubule
and water
plasma
from increased
mean
renal
arterial
pressure
The fraction of The cortical and The
calculated
PAH filtered is medullary
clearance of PAH
less
than
the collecting
ducts depends on the
filtration fraction
are
able
to urinary flow rate
reabsorb
some
PAH
Descending limb
of the loop of
Henle
The distal tubule
has
a
thicker
basement
membrane
The efferent and
afferent arterioles
are
both
constricted
Ascending limb of Distal tubule
the loop of Henle
The
proximal
tubule
has
a
thicker basement
membrane
Only the afferent
arteriole
is
constricted
The
proximal
tubule forms the
juxtaglomerular
apparatus
Only the efferent
arteriole
is
constricted
Beta-adrenergic
agonists
Prostaglandins
Aldosterone
An
increased An
increased
synthesis of urea reabsorption
of
by the liver
urea
by
the
proximal tubules
It produces its It produces its It
causes
an
effect
by effect
by increased
increasing distal activating cAMP
reabsorption
of
tubular
hydrogen ion
permeability
to
sodium
Increase
the Increase
the Increase
the
permeability of the glomerular
excretion of Na
distal nephron to filtration rate
water
Adenosine
ADH
Aldosterone
A
decreased
secretion of urea
by
the
distal
tubules
It has its main
effect
on
the
proximal tubule
angiotensin II
The
measured
value
of
the
plasma
PAH
concentration
is
less than the actual
PAH
concentration
Collecting duct
The distal tubule
has fewer tight
intercellular
junctions
The
afferent
arteriole
is
constricted and the
efferent arteriole is
dilated
Stimulation of the
macula densa
An increased renal
blood flow
It is secreted in
response to an
increase in blood
pressure
Increase
the Increase
excretion of water diameter of
renal artery
Renin
the
the
Angiotensinogen
815.
12
816.
12
817.
12
818.
12
819.
12
820.
12
821.
12
822.
12
823.
12
824.
12
825.
12
826.
12
827.
12
The filtration fraction is increased by
Increasing efferent Increasing renal Increasing afferent Increasing plasma
arteriolar
blood flow
arteriolar
oncotic pressure
resistance
resistance
Which one of the following substances causes renal Adenosine
Nitric oxide
Bradykinin
Prostaglandins
blood flow to decrease?
What percentage of the filtered load of sodium is 65%
15%
25%
45%
reabsorbed by the proximal tubule?
ANP (atrial natruretic hormone) decreases Na The
inner The
proximal The
thick The
distal
reabsorption within
medullary
tubule
ascending limb of convoluted tubule
collecting duct
Henle’s loop
Which of the following can be determined by Effective
renal ECF volume
Glomerular
Plasma volume
calculating the clearance of para-aminohippuric acid plasma
flow
filtration
rate
(PAH)?
(ERPF)
(GFR)
If the plasma concentration of a freely filterable 200 mL/min
50 mL/min
125 mL/min
150 mL/min
substance that is neither secreted or reabsorbed is 0, 125
mg/mL, its urine concentration 25 mg/mL, and urine
formation 1, 0 mL/min, the glomerular filtration rate is:
The consumption of oxygen by the kidney:
Directly reflects Decreases as blood Is regulated by Remains constant
the
level
of flow increases
erythropoietin
as blood flow
sodium transport
increases
Major determinants of plasma osmolarity include all the Hemoglobin
Sodium
Chloride
Albumin
following except:
Metabolic alkalosis can be caused by all the following Hyperaldosteronis Hyperventilation
Hypokalemia
Volume depletion
except:
m
An increase in the concentration of plasma potassium Secretion
of Release of renin
Secretion of ADH Release
of
causes an increase in:
aldosterone
natriuretic
hormone
A healthy 20-year-old man deprived of water for several Increased plasma Decreased plasma Decreased plasma Increased
days has an arterial pressure of 118/78 mm Hg and a osmolality
aldosterone
renin activity
extracellular fluid
plasma concentration of antidiuretic hormone (ADH) 5
volume
times above normal. Which of the following is the most
likely explanation for the increase in ADH
concentration?
A healthy 38-year-old woman is found unconscious and Proximal tubule
Cortical collecting Distal tubules
Loops of Henle
severely dehydrated. Her plasma levels of antidiuretic
duct
hormone (ADH) are increased about 5-fold above
normal In which portion of her kidney tubule is most of
the water being reabsorbed?
Glomerular hydrostatic pressure = 44 mm Hg, 7 mm Hg
5 mm Hg
25 mm Hg
63 mm Hg
Bowman's capsule hydrostatic pressure = 9 mm Hg,
Osmotic pressure of plasma = 28 mm Hg, Osmotic
pressure of tubular fluid = 0. Given this data, what is the
Increasing
the
pressure
within
Bowman’s capsule
Dopamine
95%
The
cortical
collecting duct
Total body water
(TBW)
362 mL/min
Is greatest in the
medulla
Glucose
Vomiting
Production
angiotensin II
of
Increased
left
atrial pressure
Medullary
collecting duct
81 mm Hg
828.
12
829.
12
830.
12
831.
12
832.
12
833.
12
834.
12
835.
12
836.
12
837.
12
net filtration pressure at the glomerulus?
The ratio of urinary concentration to plasma
concentration of inulin {(U/P) inulin} decreases Which
of the following is true if the glomerular filtration rate
remains constant?
A healthy adult participating in a clinical research study
increases his daily sodium intake greatly, but his plasma
sodium remains at a constant level. Which of the
following substances is most responsible for this
constancy in plasma sodium concentration when large
amounts of sodium are ingested?
A certain substance, which is both freely filtered and
secreted, is being maximally secreted. As the plasma
concentration of the substance increases, the renal
clearance:
A hypertensive patient is found to have a partial
obstruction of the renal artery due to an atherosclerotic
plaque. The resultant decrease in blood flow causes the
increased release of an enzyme from which of the
following structures?
The syndrome of inappropriate antidiuretic hormone
secretion (SIADH) is caused by the excess release of
ADH. SIADH will have the LEAST effect on:
Amino acids are almost completely reabsorbed from the
glomerular filtrate via active transport in the:
The daily production of hydrogen ion from CO2 is
primarily buffered by:
A freely filterable substance that is neither reabsorbed
nor secreted has a renal artery concentration of 12
mg/mL and a renal vein concentration of 9 mg/mL.
Calculate the filtration fraction (GFR/RPF)
All the following comparisons between the distal
nephron and the proximal tubule are correct except one.
Find out the exception
The pH of the tubular fluid in the distal nephron can be
lower than that in the proximal tubule because:
Urine flow rate Aldosterone levels Inulin
clearance Positive free water Reabsorption
of
has increased
have increased
has decreased
clearance
has inulin has increase
decreased
Antidiuretic
hormone
Aldosterone
Angiotensin II
Atrial natriuretic Epinephrine
factor (ANF)
Decreases
and Sncreases
and increases to the will decrease to will remain
approaches that of approaches that of renal plasma flow zero
same
inulin
inulin
Juxtaglomerular
cells
Afferent arterioles
Urinary flow
Proximal tubule
Red blood
hemoglobin
0.25
Kupffer cells
Proximal
convoluted tubule
Concentration of Total body water
plasma sodium
Urinary osmolarity
Plasma osmolarity
Loop of Henie
Distal tubule
Collecting duct
Renal pelvis
Red blood
bicarbonate
0.15
cell Plasma proteins
cell Extracellular
bicarbonate
0.05
Arcuate arteries
the
The distal nephron
secretes
more
hydrogen ion than
does the proximal
tubule
The distal nephron
is less permeable
to hydrogen ion
than
is
the
proximal tubule
The distal nephron
is more responsive
to aldosterone than
is the proximal
tubule
The tight junctions
of
the
distal
nephron are less
leaky to solute
A greater sodium
gradient can be
established across
the wall of the
More buffer is
present in the
tubular fluid of the
distal nephron than
Plasma phosphate
0.35
0.45
The distal nephron
has
a
more
negative
intraluminal
potential than does
the
proximal
tubule
More
hydrogen
ion is secreted into
the distal nephron
than
into
the
The distal nephron
secretes
more
potassium
than
does the proximal
tubule
The brush border
of
the
distal
nephron contains
more
carbonic
838.
12
839.
12
840.
12
841.
12
842.
12
843.
12
844.
12
845.
12
846.
12
847.
12
848.
12
Which of the following statements about renin is true?
than those of the distal nephron than
proximal tubule
across the wall of
the
proximal
tubule
It
converts It is secreted by
angiotensinogen to cells
of
the
angiotensin I
proximal tubule
in the
tubule
proximal proximal tubule
Its secretion leads
to loss of sodium
and water from
plasma
Its secretion is
stimulated
by
increased
mean
renal
arterial
pressure
H+ secretion in the distal nephron is enhanced by all the Hyperkalemia
An increase in the An increase in the Metabolic acidosis
following except one. Find out the exception:
level of plasma tubular lu-minal
aldosterone
concentration of
poorly
reabsorbable
anions
In controlling the synthesis and secretion of aldosterone, Adrenocorticotropi Renin
Angiotensin II
Concentration of
which of the following factors is least important?
c
hormone
plasma Na+
(ACTH)
Urinary volume is increased by all the following except Sympathetic
Diabetes insipidus Diabetes mellitus
Increased
renal
one Find out the exception:
stimulation
arterial pressure
Aldosterone causes a decrease in the renal tubular Sodium
Magnesium
Hydrogen
Potassium
secretion of:
The amount of potassium excreted by the kidney will Circulating
Distal tubular flow Dietary intake of Na+ reabsorption
decrease if:
aldosterone levels increases
potassium
by
the
distal
increase
increases
nephron decreases
Which of the following substances will be more Creatinine
Glucose
Sodium
Bicarbonate
concentrated at the end of the proximal tubule than at
the beginning of the proximal tubule?
When a person is dehydrated, hypotonic fluid will be Proximal tubule
Glomerular filtrate Loop of Henie
Distal convoluted
found in the:
tubule
The electrically neutral active transport of sodium from Ascending limb of Proximal tubule
Descending limb Cortical collecting
the lumen of the kidney occurs in the:
the loop of Henie
of the loop of duct
Henie
Decreasing the resistance of the afferent arteriole in the The renal blood The renal plasma The
filtration The
oncotic
glomerulus of the kidney will decrease
flow
flow
fraction
pressure of the
peri-tubular
capillary blood
If GFR increases, proximal tubular reabsorption of salt An increase in An increase in A decrease in An increase in
and water will increase by a process called peritubular oncotic peritubular
peritubular sodium proximal tubular
glomerulotubular balance. Contributions to this process pressure
capillary
concentration
flow
include
hydrostatic
pressure
anhydrase
than
that
of
the
proximal tubule
It
converts
angiotensin I to
angiotensin II
Respiratory
acidosis
Concentration
plasma K+
of
Infusion
mannitol
Ammonium
of
The excretion of
organic
ions
increases
Phosphate
Collecting duct
Medullary
collecting duct
The
glomerular
filtration rate
An increase
peritubular
capillary flow
in
849.
12
850.
12
851.
12
852.
12
853.
12
The glomerular filtration barrier is composed of all the
following except one Find out the exception
Fenestrated
capillary
endothelium
Renin release from the juxtaglomerular apparatus is Increased pressure Beta-adrenergic
inhibited by:
within the afferent agonists
arterioles
Potassium secretion into the urine is controlled by:
aldosterone
renal tubular cells
secretion
The capillary beds that receive blood from the efferent glomerular
nephron capillaries
arterioles (іn the kidney) are called the:
capillaries
If a substance (x) is cleared by the kidney at a rate Actively secreted
Freely filtered
greater than inulin, it must be:
Basement
membrane
Podocytes
Mesangial cells
Prostaglandins
Aldosterone
Stimulation of the
macula densa
renin production
ADH
Growth hormone
peritubular
capillaries
Freely
filtered+
reabsorbed
pulmonary
capillaries
Actively
reabsorbed
coronary
capillaries
Actively
reabsorbed
secreted
Ureteric
obstruction
854.
12
Increase in GFR occurs with:
855.
12
856.
12
857.
12
858.
12
The primary event leading to the reabsorption of water
in the kidney is the active reabsorption of:
Which of the following is generally infilterable through
the glomeruli?
Calculate the effective glomerular filtration pressure
from the following data:Average glomerular hydrostatic
pressure = 78 colloid osmotic pressure of glomerular
blood = 28 hydrostatic pressure of Bowman's capsule
=14
If the arterial blood pressure in the glomerulus is
approximately 50 mmHg, what will happen to alter the
situation for this person?
859.
12
860.
12
861.
12
Macula densa
Decreased
sympathetic
stimulation
Sodium
Increased
sympathetic
stimulation
Water
Decreased
blood flow
Chloride
Potassium
Magnesium
Fibrinogen
Glucose
Sodium
Fatty acids
Magnesium
36 mm Hg
22 mm Hg
19 mm Hg
14 mm Hg
18 mm Hg
waste
products
such as urea and
creatine
will
decreased in his
blood
sodium ions will
be
excreted
leading
to
electrolyte
imbalance
a
reflex
vasodilation of the
afferent arteriole
will
occur
to
increase the rate of
filtration at the
glomerulus
If GFR increases, proximal tubular reabsorption of salt An increase in
and water will increase by a process called proximal tubular
glomerulotubular balance. Contributions to this process flow
include:
The filtered HCO3- within the proximal tubular lumen is CO2
reabsorbed mainly in the form of:
Which of the following substances has the lowest Glucose
clearance in the human kidney:
An increase
peritubular
capillary
hydrostatic
pressure
H2CO3
Creatinine
renal Hypoproteinaemia
a
reflex
vasoconstriction of
the
afferent
arteriole will occur
to increase the rate
of filtration at the
glomerulus
in A decrease in An increase in
peritubular sodium peritubular oncotic
concentration
pressure
+
GFR
will
increased
be
An increase
peritubular
capillary flow
in
H+
HCO3-
OH-
PAH
K+
Cl-
862.
12
863.
12
864.
12
865.
12
866.
12
867.
12
868.
12
869.
12
870.
12
If a substance is transported from the plasma of the
peritubular capillary into the fluid of the renal tubule, it
is said to be:
Active reabsorption of glucose appears to occur in the:
Secreted
Proximal
convoluted tubule
How many liters of glomerular filtrate per day is 180
produced by kidneys under normal circumstances?
Potassium secretion into the filtrate and eventual urine:
Is associated with
sodium
reabsorption from
the
distal
convoluted tubules
If the arteriole that supplies blood to the glomerulus The
glomerular
becomes constricted:
filtration
rate
decreases
What percentage of the blood circulating through the
body makes it through the kidneys in one day?
Given the following conditions, glomerular blood
hydrostatic pressure of 75 mmhg, capsular hydrostatic
pressure of 15 mmhg, blood osmotic pressure of 25
mmhg, capsular osmotic pressure of 0 mmhg, the
effective filtration pressure would be?(mmHg):
A woman has a respiratory rate of 18, a tidal volume of
350 mL, and a dead space of 100 mL. What is her
alveolar ventilation?
When the respiratory muscles are relaxed, the lungs are
at
Filtered by blood Excreted
pressure
osmosis
Henle's loop
1500
by Reabsorbed
Distal convoluted Collecting tubule
tubule
60
80-120
Filtered
reabsorbed
+
Peritubular
capillary
100
is decreased by the Is
controlled Is usually low Doesn't
occur
potassium
- primarily by ADH because potassium because potassium
sparing hormone
is low in the diet
is the chief ion
inside cells
25%
Blood flow into Hydrostatic
the
efferent pressure in
arteriole increases glomerulus
increases
15%
40%
35
40
65
25
0
4.5 L
4.0 L
5.0 L
5.5 L
6.0 L
Functional
Residual
residual capacity (RV)
(FRC)
The
protein
the concentration of
the
filtrate
increases
75%
The
protein
concentration of
the
filtrate
decreases
100%
volume Expiratory reserve Inspiratory reserve Total
lung
volume (ERV)
volume (IRV)
capacity (TLC)